BR Real Property

Pataasin ang iyong marka sa homework at exams ngayon gamit ang Quizwiz!

• When a developer subdivides land into several parcels and some of the deeds contain negative covenants but some do not, negative covenants or equitable servitudes binding all the parcels in the subdivision may be implied under the doctrine of "reciprocal negative servitudes." Two requirements must be met before reciprocal negative servitudes will be implied:

(i) a common scheme for development, and (ii) notice of the covenants. The second requirement may be satisfied by actual notice, record notice, or inquiry notice.

What are the different ways an easement might be created:

1) Express grant or reservation (SOF applies) - Note that an oral grant just creates a license, which isn't an interest in land 2) Implication - by operation of law. This can occur in two ways: a) By use existing before a tract was divided b) By necessity for a landlocked parcel 3. Presumption - acquired through adverse, open and notorious, and continuous use for the statutory period

A court won't enforce an equitable servitude if:

1) The neighborhood conditions have changed so significantly that enforcement would be inequitable. What's never good enough here is piecemeal change or mere pockets of limited change. 2) The person seeking enforcement is violating a similar restriction on his own land (unclean hands); 3) A benefited party acquiesced in a violation of the servitude by a burdened party; 4) A benefited party acted in such a way that a reasonable person would believe the covenant was abandoned or waived (estoppel); or 5) The benefited party fails to bring suit against the violator within a reasonable time (laches)

1. A landowner has a right to have her land supported in its natural state by adjoining land. If, however, the land has buildings on it, an excavating neighbor is liable for damage to the buildings caused by the excavation if: (i) the excavating landowner was negligent; or (ii) the excavation would have caused the land to subside even in its natural state (i.e., without buildings). Here, the facts indicate that the developer's excavation caused the damage to the boutique owner's shop. 2. Regarding the running of a real covenant and the requirement of notice: in common law js, a purchaser of land that was subject to a covenant takes the land burdened by the covenant - it doesn't matter whether the purchaser had notice of the covenant prior to purchasing. However, in recording statute js, if the covenant isn't recorded, a BFP who has no notice of the covenant and records his own deed will take possession of the land free of the covenant. Most states don't require closings to be conducted by an attorney. But if a non-attorney does it, he can't offer legal advice as though he was an attorney

1. A life tenant is obligated to pay all ordinary taxes on the land to the extent of the income or profits derived from the land. However, if the life tenant is in possession of the land himself, he is liable for the taxes and other carrying charges to the extent of the greater of the economic benefits he derives from the land or the land's reasonable rental value. Here, the partner lived on the land. It doesn't appear that the partner is deriving any profits from the land; so, he is obligated to pay taxes for his period of occupancy to the extent of the reasonable rental value. Based on the prior offers, the reasonable rental value seems to be between $15,000 and $20,000 per year, which was more than the amount of the taxes. Failure to pay the taxes is permissive waste, and a future interest holder who expends funds in satisfaction of the life tenant's obligation is entitled to reimbursement. Thus, the friend is entitled to reimbursement of the amount he paid. 2. In a partial condemnation case, the landlord-tenant relationship continues, as does the tenant's obligation to pay the entire rent for the remaining lease term. 3. the covenant of quiet enjoyment can be breached only by actions of the landlord and not those of a third party, such as the government.

When closing occurs, the k merges with the deed and disappears. The seller is no longer liable under the k, only under the deed. To pass legal title from grantor to grantee, deed must: 1. Be lawfully executed, and 2. Delivered. Executing a valid deed requires the following:

1. A writing signed by the grantor 2. An unambiguous description of the land. This description doesn't have to be perfect. But if the description is insufficient to provide "a good lead," title isn't transferred, and grantor retains title. If a description is just ambiguous, rather than inadequate, parol evidence is allowed in to clear up ambiguity 3. Identification of the parties by name or description 4. Words of intent to transfer, such as "grant"

There are four main types of easements:

1. Affirmative easement - right to use someone else's land 2. Negative easement - right to prevent something on another's land 3. Easement appurtenant - involves two tracts of land 1) Dominant parcel has the benefit, which runs to grantees 2) Servient parcel has the burden, which runs to grantees with notice 4. Easement in gross - involves one tract of land

1. A disabling restraint is one that renders any attempted transfer ineffective. All disabling restraints on legal interests (e.g., a fee simple or life estate) are void. Here, the son's life estate, a legal interest, is subject to a disabling restraint prohibiting him from transferring his interest. The restraint is void, and the son's conveyance to the friend is valid. The friend takes what the son had: an estate for the son's life. In contrast to a disabling restraint, a forfeiture restraint on alienation, under which an attempted transfer forfeits the interest, is valid if it is limited as to time and reasonable. Here, the restraint is limited to the time before the interest becomes possessory and seems reasonable; so, any attempt by the granddaughter or grandson to transfer their interest while the interest was still a remainder would cause a forfeiture. Thus, the grandson's attempted transfer to the neighbor was invalid and caused a forfeiture of the grandson's remainder in favor of the granddaughter

1. Although it is difficult for a co-tenant to adversely possess against other co-tenants, a co-tenant can establish title by adverse possession if her possession is hostile to other co-tenants. To satisfy this requirement, she must either physically oust them from the property or constructively oust by informing them that she is asserting exclusive control over the property. 2. To extinguish an easement by prescription, the owner of the servient tenement must so interfere with the easement as to create a cause of action in favor of the easement holder. The interference must be open, notorious, continuous, and nonpermissive for the prescriptive period. 3. An intent to have a transfer be effective at the grantor's death is valid in a will, but not a deed, unless the deed expressly reserves a life estate. If it doesn't do that, there has been no deed delivery. 4. After foreclosure begins, but before the foreclosure sale occurs, mortgagor has the equitable right of redemption.

What types of security interests may a party have in land other than a mortgage:

1. Deed of trust—similar to a mortgage, but a third-party trustee forecloses 2. Installment land k—seller retains the deed until buyer pays in full 3. Absolute deed—treated as an equitable mortgage when given for a debt 4. Sale-leaseback—ct may determine this was a disguised mortgage

What are the nonpossessory types of interests that may exist in property:

1. Easements 2. Profits Real Covenants 4. Equitable servitudes

The order of priority for allocating mortgage foreclosure sale proceeds is as follows, from first to last:

1. Expenses of the sale, including attorneys' fees, and court costs; 2. The principal and accrued interest on the foreclosing party's loan; 3. Any junior lienors in the order of their priority; and 4. The mortgagor. In many cases, no surplus remains after the principal debt is paid off. Senior lienors receive none of the proceeds. Bc a senior lien remains on the property (i.e., may itself be foreclosed in the future), a senior lienor is not entitled to any of the money from the sale, even if there is a surplus.

A remainder is contingent if:

1. It's created in unborn or unascertained persons, or a) Exs: A) O conveys "to A for life, then to B's heirs." A is alive. B is alive. Bc a living person has no heirs, while B is alive, his heirs are unknown. Thus, the remainder is contingent. B) O conveys "to A for life, then to those children of B who survive A." A is alive, as is B. But we don't yet know which, if any, of B's kids will survive A. The remainder in B's kids is contingent. 2. It's subject to a condition precedent, or a) A condition is precedent if it must be satisfied before the remainderman has right to possession. b) Look for the condition to appear BEFORE the language creating the remainder or for it to be woven into the grant to the remainderman. c) Exs A) O conveys "to A for life, then to B and his heirs if B marries C." B's remainder is contingent bc he must marry C before he can take possession. B) O conveys "to A for life, then to B and his heirs if B marries C, otherwise to D and his heirs." B and D have alternative contingent remainders. 3. Both. Aka, a remainder may be contingent as to a person or as to an event

What are the three types of concurrent interests that parties might have in land (aka, three types of co-ownership):

1. Joint tenants (JTs) - two or more co-tenants with rights of survivorship (ie, dead co-tenant's share passes to the remaining co-tenants). Created expressly, severed by a tenant's sale or suit for partition. 2. Tenants by the entirety - two spouses with rights of survivorship. Created expressly or presumed in some states by a grant to spouses, severed by divorce 3. Tenants in common - two or more co-tenants, no right of survivorship. Created by the severance of the above tenancies

A fixture is a chattel that's been affixed to land and has become part of it. It passes with the land's ownership and must stay put. Exs: plumbing, AC. This gets complicated when different people own the land and fixtures:

1. Landlord - tenant: An agreement between the landlord and tenant is controlling on whether an annexed chattel is a fixture. Absent an agreement, a tenant is deemed to lack the intent to permanently improve the property, and thus may remove his annexed chattels if removal does not substantially damage the premises or destroy the chattel. Annexed chattels must be removed by the end of the lease term (or n a reasonable time after the termination of an indefinite tenancy), and tenant is responsible for repairing any damage caused by the removal. 2. Life tenant and remaindermen: same rules apply as in landlord-tenant situation, except life tenant's representative can remove annexations in a reasonable time after the tenant's death 3. Licensee or trespasser and landowner: licensees are treated like tenants, but trespassers lose their annexations. Thus an adverse possessor or good faith trespasser can't remove fixtures. However, some cts allow a good faith trespasser recovery measured by the value they added to the land.

What are the three theories of title (in respect to mortgages):

1. Lien theory - mortgagee holds a security interest only 2. Title theory - mortgagee holds title until mortgage is satisfied 3. Intermediate theory - mortgagee holds title only after default

Surface Waters: Surface water is water without a channel that passes over land, such as water coming from rain, springs, or melting snow, which has not yet reached a natural watercourse or basin. A landowner can use surface water within their boundaries for any purpose they desire. Questions on surface water usually concern liability for changing natural flow by dikes, drains, and so on. Liability depends on which theory the state follows. Remember that these theories apply to redirecting surface water. A landowner can capture (for ex, by a dam or in barrels) as much surface water as they wish. Surface water can be diverted to any purpose on or off the land. Owners below have no cause of action unless the diversion was malicious.

1. Natural Flow Theory - Followed by many states. Owners cannot alter natural drainage patterns. This rule has been "softened" in most states to allow "reasonable changes." 2. Common Enemy Theory - Under this theory, followed by many states, an owner can take any protective measures to get rid of the water or combat its flow (ex, dikes or drainage changes). The rule has been modified by many courts to prohibit unnecessary damage to others' lands. 3. Reasonable Use Theory - There is a growing trend to apply this theory, which involves balancing the utility of the use against the gravity of the harm.

Two promises that are implied in every land sale k:

1. Seller won't make false statements of material fact 2. Seller will provide marketable title

To create a valid death escrow, grantor has to place the deed beyond his control and reserve no power over it.

1. The common law rule that priority is given to the grantee who was first in time still applies unless operation of the jurisdiction's recording statute changes the result. The statute in this question is a race-notice statute, under which a subsequent BFP is protected only if she records her interest before the prior grantee does. While the developer is a bona fide purchaser, she must still win the race to the recording office to prevail over the prospector's prior interest. Since neither party has recorded in this fact pattern, the developer will take the property subject to the prospector's prior interest. 2. Warranty of title isn't breached by a grantor's refusal to defend title against a wrongful claim by a third party.

Tenant has two primary duties under a lease:

1. To repair 2. To pay rent One additional duty: Tenant's duty to not use premises for illegal purpose

Elements of the common scheme doctrine:

1. When the sales began, the subdivider (A) had a general scheme of residential development which included the defendant's lot (the scheme may be evidenced by: a recorded plat, a general pattern of restrictions, or oral representations to early buyers); and 2. The defendant lot-holder (B) had notice of the promise contained in those prior deeds when it took. Notice can come in one of three forms: 1) Actual notice, meaning the defendant had literal knowledge of the promises contained in the prior deeds 2) Inquiry notice, meaning the neighborhood seems to conform to the common restriction (it's the "the lay of the land") 3) Record notice, meaning the form of notice sometimes imputed to buyers on the basis of the publicly recorded documents

Requirements for the benefit of a covenant to run with the land (from B to B-1): (WITV)

1. Writing - og promise between A and B must've been in writing 2. Intent - A and B must've intended the benefit would run 3. Touch and concern - benefit touches and concerns the land if the promised performance benefits the promisee. 4. Vertical privity - non-hostile nexus between B and B-1. Note that Horizontal privity is not required for the benefit to run. Where horizontal privity is lacking, B-1 can enforce the covenant against A, but not A-1. Generally, promises to pay money (HOA fees), and covenants not to compete, run with the land.

D. The different types of deeds (aka, covenants for title) - general warranty, special warranty, quitclaim deed 1. Quitclaim deed - conveys only what the grantor had at the time of conveyance 2. General warranty deed a. Warrants against all defects in title, including those attributable to grantor's predecessors b. This type of deed generally contains all six of the following covenants (it is only under these covenants that a seller can be held liable, under the deed, after closing): 1) Present covenants - these covenants are breached, if ever, at the time the deed is delivered a) The covenant of seisin b) The covenant of the right to convey c) The covenant against encumbrances A) Note that the rule regarding visible and known encumbrances is different for the real estate k and the deed (post-closing). For real estate ks, remember that a purchaser is generally presumed to have contracted to accept the land subject to visible easements. For deeds, remember that most js hold that the covenant against encumbrances is breached even if the grantee knew of the encumbrance (whether or not it is visible).

2) Future covenants - not breached, if ever, til the grantee is disturbed in possession a. Covenant for quiet enjoyment: grantor promises that grantee won't be disturbed in possession by a third party's lawful claim of title b. Covenant of warranty: grantor promises to defend against reasonable claims of title by a third party and to compensate the grantee for any loss caused by the superior title's claim c. Covenant for further assurances: grantor promises to do whatever is needed to perfect grantee's title if it later turns out to be imperfect 3. Special warranty deed a. Contains the same covenants as the general warranty deed, but the grantor here makes those promises only on behalf of himself, NOT for his predecessors in interests

7. Vested remainders a. A vested remainder is one created in an existing and ascertained person, and not subject to a condition precedent b. There are three types of vested remainders: 1) The indefeasibly vested remainder a) Is transferable, descendible, and devisable. b) Not subject to RAP c) Ex: To A for life, then to B d) A vested remainder that's not subject to divestment or diminution. The holder of this remainder is certain to acquire an estate in the future, with no strings or conditions attached.

2) The vested remainder subject to total divestment/subject to complete defeasance a) Ex: To A for life, and on A's death, to B; but if B predeceases A, then to C (B has a vested remainder subject to total divestment) b) Is transferable, descendible, and devisable. c) Isn't subject to RAP d) A vested remainder that's subject to a condition subsequent. e) Here, the remainderman exists. His taking isn't subject to any condition precedent. However, his right to possession could be cut short bc of a condition subsequent f) Ex: O conveys "to A for life, then to B and his heirs; but if B dies unmarried, then to C and his heirs." B is alive and unmarried. B has a vested remainder subject to total divestment by C's executory interest. g) Distinguishing conditions subsequent and precedent A) It's important to know the difference between a condition precedent, which creates a contingent remainder, and a condition subsequent, which creates a vested remainder subject to total divestment. To tell the difference, apply the Comma Rule. When conditional language in a transfer follows language that, taken alone and set off by commas, would create a vested remainder, the condition is a condition subsequent, and you have a vested remainder subject to complete defeasance. B) Note that where language is ambiguous, the preference is for vested remainders subject to divestment rather than contingent remainders or executory interests

E. Foreclosure 1. A mortgagee can foreclose by a proper judicial proceeding. At foreclosure, the land is sold, and the sale proceeds go to satisfying the debt. 2. Deed in lieu of foreclosure: a. Mortgagor may tender to the mortgagee a deed in lieu of foreclosure, which permits the mortgagee to take immediate possession without a foreclosure sale. 1) Note that this won't terminate any junior liens on the land, bc not technically a foreclosure 3. If the proceeds from the foreclosure sale are less than the amount owed, junior liens are paid off in order of their priority. Any remaining surplus goes to the DO.

4. Effect of foreclosure on various interests a. After foreclosure, all interests senior to the foreclosed interest remain on the property, and all interests junior to it are extinguished. b. Junior lienholders will be paid in descending order with the proceeds from the sale, assuming funds are left over after full satisfaction of superior claims. Junior lienholders should be able to proceed for a deficiency judgment. But once foreclosure of a superior claim has occurred, with the proceeds distributed appropriately, junior lienholders can no longer look to the land for satisfaction. 5. Necessary parties a. The debtor-mortgagor is considered a necessary party and must be joined, particularly if the creditor wishes to proceed against the debtor for a personal deficiency judgment. b. Failure to include a necessary party results in the preservation of that party's claim, despite the foreclosure and sale. Thus, if a necessary party is not joined, their mortgage will remain on the land.

3. Real covenants a. Creation: writing signed by grantor b. Burden of promise will run to successor if WITHN: 1) Writing 2) Intent 3) Touch and concern 4) Horizontal and vertical privity 5) Notice c. Benefit of promise will run to sin grouccessor of benefitted lot if WITV: 1) Writing 2) Intent 3) Touch and concern 4) Vertical privity d. Remedy: ds

4. Equitable servitudes a. Creation: writing signed by grantor (unless implied by General Scheme Doctrine) b. Successors bound if WITNes: 1) Writing 2) Intent 3) Touch and concern 4) Notice (privity not required) c. Remedy: ds 5. Reciprocal negative servitudes (Common Scheme Doctrine) a. Creation: 1) Majority: in a subdivision, residential restrictions in prior deeds by a common grantor bind subsequent grantees whose deeds don't have the restriction if, at the start of the subdivision: a) Grantor had common scheme, and b) Unrestricted lot holders had notice 2) Minority: not binding on subsequent grantees unless their lots are expressly restricted in writing b. Where common scheme exists, subsequent purchasers with notice are bound c. Remedy: injunction

A co-tenant must not commit waste on the property; if they do commit waste, one of their co-tenants can bring an action against them. There are three types of waste: 1) Voluntary waste = willful destruction 2) Permissive waste = neglect 3) Ameliorative waste = unilateral change that increases value A JT or a TIC can bring an action for partition, to divide the concurrent ownership of the land. There are three kinds of partition: 1) Voluntary agreement 2) Partition in kind 3) Forced sale Cts prefer partition in kind, but will permit partition by sale when a fair and equitable physical division can't be made. Note that restrains on partition are valid provided they're limited to a reasonable time.

A JT or TIC may encumber their interest through a mortgage or lien, but they can't encumber the other co-tenant's interests. If a JT is involved, then in a lien theory state, a mortgage or lien doesn't sever the JT, but a foreclosure sale will. Note that in the case of a JT, if the obligated co-tenant dies before foreclosure, the mortgagee's interest will extinguish, bc that interest will end.

A profit is a right to take resources from another's land. Ex: O allows A to come onto O's land to cut and remove timber. A writing is required to create a profit. In terms of termination, they can be terminated in the same ways an easement can be terminated.

A Real covenant/equitable servitude is a promise to do, or not do, something on the land. Ex: O conveys an adjoining parcel to A. A promises not to build a swimming pool on the property. A writing is required for their creation. The exception to this: an equitable servitude may be implied from a common scheme of development of residential subdivision Also, equitable defenses may apply to enforcement of servitude

Every conveyance of real estate has a two step process: 1. The land k, which conveys equitable title; and 2. The closing, where the deed passes legal title and becomes the operative document.. Prior to closing, k law rules; after closing, real property law controls.

A land sale k must be in writing to satisfy the SOF. This writing must: 1. Id the parties 2. Describe the property 3. Include the price or a means of determining the price (like fmv, as determined by appraisal) If the writing doesn't satisfy the SOF, buyer can enforce an oral real estate k with specific performance by showing the equitable doctrine of part performance. To show part performance, buyer must prove two of these three actions: 1. Buyer has possession of the property 2. Buyer has paid the purchase price or a significant portion of the purchase price 3. Buyer has made substantial improvements to the premises

What is a Profit:

A profit is a right to enter another's land to remove products of the soil. The profit entitles its holder to enter the servient land and take from it some resources, for example, the soil, some substance of the soil (such as minerals, timber, or oil), or some product of the property (such as fish or game). All of the rules governing creation, alienation, and termination of easements are applicable to profits. In addition, a profit may be extinguished through surcharge (misuse that overly burdens the servient estate).

Party walls and common driveways - Cts will treat a wall erected partly on the property of each of two adjoining landowners as belonging to each owner to the extent it rests upon their land. Courts will also imply mutual cross-easements of support, with the result that each party can use the wall or driveway and neither party can unilaterally destroy it. How this is created:

A written agreement is required by SOF for the express creation of a party wall or common driveway agreement, but an "irrevocable license" can arise from detrimental reliance on a parol agreement. Party walls and common driveways can also result from implication or prescription. If party wall or common driveway owners agree to be mutually responsible for maintaining the wall or driveway, the burdens and benefits of those promises run to the successive owners of each parcel.

Leases - a lease is a k that governs the landlord-tenant relationship 1. Options to purchase - are sometimes attached to leases a. As long as the option is contained within the lease itself, the consideration for the lease supports the option b. Absent a contrary provision, the option lasts as long as the lease. The method of exercise is determined by the agreement.

An easement is deemed appurtenant when the right of special use benefits the easement holder in her physical use or enjoyment of another tract of land. The land subject to the easement is the servient tenement, while the land having the benefit of the easement is the dominant tenement. The benefit of an easement appurtenant passes with transfers of the benefited land, regardless of whether the easement is mentioned in the conveyance. All who possess or subsequently succeed to title to the dominant tenement are entitled to the benefit of the easement.

I. Present estates and future interests A. Present possessory interests 1. Indefeasible interests - not subject to early termination a. Fee simple absolute (to A and his heirs; to A) b. Life estate (to A for life; to A for the life of B) 2. Defeasible interests - allows a fee simple or life estate to be terminated if a stated event occurs a. Determinable (for so long as; until; while; during) - automatically reverts to the grantor b. Subject to the condition subsequent (but if; upon the condition that; provided that) - subject to the grantor's right of entry, which must b exercised c. Subject to an executory interest (to A for so long as..., and if not..., to B; to A, but if..., to B) - divests in favor of a third party

B. Future possessory interests 1. Interests retained by the grantor a. Reversion—grantor transfers a shorter estate than she owns (grantor with a fee simple transfers a life estate) b. Possibility of reverter—grantor transfers a determinable estate c. Right of entry (power of termination)—reserved on the grant of an estate subject to a condition subsequent 2. Interests created in a transferee (Note: RAP may apply) a. Executory interests—cut short the prior estate b. Remainders—possessory only on the natural termination of the prior estate (e.g., death of the life tenant) 1) Remainders are vested if made in an ascertained person and with no conditions precedent; otherwise are contingent c. Class gifts—remainders in a class are contingent if no member of the class yet exists, vested if all possible members exist, and vested subject to open if more members might come to exist 1) Under the rule of convenience, an open class closes when any member can demand possession

A Landlord has three main duties: 1. Duty to deliver possession (aka, has to put the tenant in actual, physical possession of the premises) 2. Implied covenant of quiet enjoyment 3. Implied warranty of habitability

B. Implied covenant of quiet enjoyment 1. Arises in every lease - provides that tenant has a right to quiet use and enjoyment of the premises, without interference from the LL or a paramount title holder (ex a prior mortgagee that forecloses 2. Breach by wrongful eviction a. Two instances when this might occur: 1) Actual eviction (excludes tenant from entire leased premises) - terminates tenant's obligation to pay rent 2) Partial eviction a) Occurs when tenant is physically excluded form only part of the premises. Partial eviction by the LL relieves tenant of obligation to pay rent for the entire premises, even though tenant continues in possession of the remainder b. LL can also breach by constructive eviction. Constructive eviction occurs when the LL's breach renders the premises unsuitable for occupancy. 1) Ex: apartment floods every time it rains. 2) Elements of constructive eviction (SING): a) Substantial Interference b) Notice c) Goodbye - tenant MUST leave 3) Tenant who's ben constructively evicted may terminate lease and get ds c. Two instances when LL breaches due to actions of other tenants: 1) LL has a duty to abate a nuisance on site 2) LL must control common areas

D. Reversions 1. A reversion is the estate left in a grantor who conveys less than what they own. Ex, O conveys "To A for life;" O has a reversion. Whenever grantor transfers an estate of lesser duration than what they started with, other than instances giving rise to a possibility of reverter or a right of entry, the future interest that arises is a reversion. 2. Is devisable, inheritable, and transferable. Its holder can sue for waste and for tortious damage to the reversionary interest. E. Future interests in transferees 1. If a future interest is held by someone other than the grantor, it has to be either: a. A contingent remainder, b. A vested remainder 1) There are three types: a) Indefeasibly vested remainder b) The vested remainder subject to complete defeasance/subject to total divestment c) The vested remainder subject to open c. An executory interest 1) There are two types: a) The shifting executory interest b) The springing executory interest

B. Remainders 1. A remainder is a future interest in a third person that can become possessory on the natural expiration of the preceding estate. It can't divest a prior estate, and it can't follow a time gap after the preceding estate. 2. A remainder must be expressly created in the instrument creating the preceding possessory estate 3. Remainders are either vested or contingent. 4. Remainders always accompany a preceding estate of known, fixed duration (like a life estate or a term of years). 5. A remainder never cuts short or divests the prior taker. 6. Contingent remainders a. Exs: 1) TO A for life, then to B if B marries C 2) To A for life, then to A's surviving kids b. Is transferable in most states (not at common law), descendible, and devisable c. Is subject to the RAP

V. Transfers of leaseholds and LLs' tort liability A. Assignments versus subleases 1. In the absence of some prohibition in the lease, a tenant may freely transfer their interest in whole (thereby accomplishing an assignment, meaning a transfer of the entire remaining term of a lease) or in part (thereby accomplishing a sublease, meaning the tenant has retained some part of the remaining term, other than a right to reenter upon breach). In the lease, a landlord can prohibit a tenant from assigning or subletting without the landlord's prior written approval. However, once a landlord consents to one transfer by a tenant, LLwaives the right to object to future transfers by that tenant unless he expressly reserved that right. 2. Covenants against assignment or sublease a. A valid covenant against assignment is considered waived if the LL was aware of the assignment and did not object (ex, by knowingly accepting rent from the assignee). Once a LL consents to one transfer by a tenant, he waives the right to object to future transfers by that tenant, unless he expressly reserves the right (known as the Rule in Dumpor's Case). Lease covenants restricting assignment and sublease are strictly construed against LL (Thus, a covenant prohibiting assignment doesn't prohibit subleasing, and vv). 1) Note: If a tenant assigns or sublets in violation of a lease provision, LL usually may terminate the lease or sue for ds, but the transfer is not void.

B. Types of assignments/subleases that may occur 1. Assignment by a landlord a. Tenant's consent isn't required. Assignee and tenant are in privity of estate, but assignee and tenant aren't in privity of k. Og ll and tenant remain in privity of k. b. Assignee is liable to tenant on all covenants that run with the land. Og LL remains liable on all covenants in the lease. 2. Assignment by a tenant a. LL's consent may be required by the lease. b. Assignee and LL are in privity of estate. Assignee and LL aren't in privity of k. Og teanant and LL remain in privity of k. c. Assignee is liable to LL on all covenants that run with the land. Og tenant remains liable for rent and all other covenants in the lease. 3. Sublease by a tenant a. LL's consent may be required by the lease. b. Sublessee and LL aren't in privity of estate. Og tenant remains in privity of estate with the LL. c. Sublessee and LL aren't in privity of k. Og tenant and LL remain in privity of k. d. Sublessee isn't personally liable on any covenants in the og lease and can't enforce the LL's covenants. Og tenant remains liable for rent and all other covenants in the lease, and can enforce the LL's covenants. e. If sublessee fails to pay rent, L proceeds against T, and T proceeds against sublessee. If the residential premises betray the implied warranty of habitability, sublessee proceeds against T, and T proceeds against L.

III. Future interests A. Future interest gives holders the right or possibility of a future possession of land that are also a legally protected right that presently exists in the property. B. Six categories of future interests - these are classified by whether they're retained by the transferor/grantor or by a transferee a. Future interests in transferor - reversionary interests: 1) Possibility of reverter 2) Right of entry 3) Reversion b. Future interests in a transferee 1) A contingent remainder 2) A vested remainder 4) An executory interest

C. Possibilities of reverter and rights of reentry 1. Possibility of reverter a. Correlative present interest: FSD b. Ex: To A so long as alcohol isn't used on the premises c. Rights of grantor: estate automatically reverts to grantor upon occurrence of stated event d. Alienability: transferable, descendible, and devisable 2. Right of entry a. Correlative present interest: Fee simple subject to a condition subsequent b. Ex: "To A on the condition that if alcohol is used on the premises, O shall have the right to reenter and retake the premises" c. Rights of grantor: Estate doesn't revert automatically; grantor must exercise his right of entry d. Alienability: descendible and devisable, but some cts hold not transferable inter vivos

RAP applies to the equitable future interests of the beneficiaries in a private trust just as it does to "legal" future interests Creation - A trust can be created by will (testamentary trust), inter vivos transfer of the trust res, or inter vivos declaration that the settlor is holding property in trust. All trusts of real property must be in writing. Note that a settlor may bequeath (by will) property to a trust created during their lifetime—that is, they may "pour it over" into the trust.

Charitable trusts: A charitable trust must have a charitable purpose. The rules governing charitable trusts differ from those applicable to private trusts in three important ways: (1) a charitable trust must have indefinite beneficiaries; (2) it may be perpetual (aka RAP doesn't apply); and (3) the cy pres doctrine, which allows a ct to select an alternative charity when the purpose of the settlor becomes impracticable or impossible, applies. Charitable trusts may be enforced by an action of the state's AG. Note: remember that the RAP DOES apply to a shift from a private to charitable use or a charitable to a private use

Many real estate closings involve signing or producing many documents. Some of the most critical closing documents:

Closing disclosure: Residential mortgage lenders must provide a closing disclosure to mortgagors at least 3 business days prior to closing. This disclosure must provide details about the mortgage, including: Principal, interest, payment amounts, closing costs, potential surprises to mortgagors like future changes in interest rates or balloon payments, and cash required to close. If mortgagee doesn't provide the disclosure, the mortgagor may be able to cancel the mortgage or get ds Notification of defects - a seller of residential property must provide a form to the buyer at closing, notifying buyer of any physical defects of which seller is aware. Seller who fails to disclose a known defect that must be disclosed in this form is liable for the defect even after closing. Environmental report: Buyers of commercial real estate often ask sellers to guarantee the property complies with environmental laws. This report ids which environmental guarantees the seller is willing to make

Distinctions between commercial and residential tenants' treatment under a tenancy at will:

Commercial tenants may be held to a new year-to-year periodic tenancy if the og lease term was for one year or more. But if the og term was less than one year, the new tenancy is typically a month-to-month tenancy. Residential tenants, however, are generally held to a new month-to-month tenancy, regardless of the og term. If the landlord notifies the tenant before the lease expires that occupancy after the termination will be at an increased rent, the tenant, by holding over, is held to have acquiesced to the new terms (even if the tenant actually objected to the new terms). Holding over equals assent.

XIII. The recording system A. Common law: first in time, first in right. B. Note, however, that a subsequent BFP is not protected by the recording acts against interests that arise by operation of law (for example, prescriptive and implied easements, or title by adverse possession). Why? Because in such instances there is no instrument to record, so the recording acts do not apply and subsequent purchasers take subject to these interests. C. The three types of recording acts: 1. In a race jurisdiction, B wins, if he records properly before A does. a. Statute's language will just include "first" 2. In a notice jurisdiction, B wins if he was a BFP when he took, regardless of whether he records before A does. In a notice state, the last BFP to take wins. a. Statute's language will just include "notice." 3. In a race-notice jurisdiction, B wins if B is a BFP and he records properly before A does. a. Statute's language includes "first" and "notice."

D. The recording statutes protect purchasers. Donees, heirs, and devisees aren't protected (unless shelter rule applies) bc they don't give value. 1. To be a BFP, a grantee must: a. Be a purchaser (not receive property by gift, will, or inheritance); b. Pay valuable consideration; c. Take without notice of the prior conveyance C. The three forms of notice a buyer potentially may be charged with: 1. Actual notice 2. Inquiry notice a. B is on inquiry notice of whatever an examination of the land would've revealed b. If someone else is in possession, is charged with inquiry notice of that fact, regardless of whether he actually inspected c. Also: if a recorded instrument makes reference to an unrecorded transaction, grantee is on inquiry notice of whatever a reasonable follow-up would've revealed 3. Record notice a. B is on record notice of A's deed if, at the time B takes, A's deed was properly recorded in the chain of title

C. Implied warranty of habitability 1. Most js imply warranty of habitability into residential leases. This warranty is not waivable. 2. This warranty provides that the premises must be fit for basic human habitation. 3. If this warranty is breached, tenant has four options (MRRR): a. Move out and terminate lease b. Repair and deduct costs from future rent c. Reduce rent or withhold all rent til ct determines fair rental value (typically, tenant must place withheld rent into escrow to show good faith) d. Remain in possession, pay full rent, affirmatively seek money ds D. Retaliatory eviction 1. In many states, LL can't terminate a lease or penalize a tenant in retaliation for exercising their legal rights a. Ex, can't raise rent if tenant reports LL for housing code violations b. Many statutes presume a retaliatory motive if LL acts within 90-180 days after tenant exercises their rights. to overcome presumption, LL has to show a valid, nonretaliatory reason for their actions

E. Anti-discrimination legislation 1. Civil Rights Act bars racial or ethnic discrimination in the sale or rental of all property 2. Fair housing act a. Protects tenants from discrimination based on race, color, religion, national origin, sex, disability, familial status b. Exemptions - doesn't apply to: 1) Owner-occupied buildings with four or fewer units in which people live independently of each other 2) Single-family homes sold or rented by an owner who owns no more than 3 single family homes c. LLs must permit disabled tenants to make reasonable modifications to existing premises to accommodate their disabilities, at the tenant's own expense. Must also make reasonable accommodations in rules, policies, services when necessary to afford them an equal opportunity to use a dwelling

Distinguishing characteristics of real covenants and equitable servitudes: 1. Real covenants a. A writing for their creation is always required. b. Running of the burden: 1) Horizontal privity (shared interest in land, apart from the covenant, by og covenanting parties, aka mortgagor-mortgagee, landlord-tenant) 2) Vertical privity (successor holds entire interest held by covenanting party c. Running of the benefit: 1) Vertical privity required d. Remedy: generally ds 2. Equitable servitudes a. Creation: writing is usually required, but they may also arise by implication from a common scheme of development of a residential subdivision b. Running of the burden: no privity required c. Running of the benefit: no privity required d. Remedy: generally an injunction

E. Breakdown of the different nonpossessory interests in land 1. Easement a. Definition: a grant of an interest in land that allows someone to use another's land b. Ex: owner of parcel A grants owner of Parcel B the right to drive across parcel A c. Is a writing required? Generally yes. Exceptions: 1) Less than one year 2) Implication 3) Necessity 4) Prescription d. Termination: 1) Stated conditions 2) Release 3) Merger 4) Abandonment 5) Estoppel 6) Prescription 7) End of necessity 2. License a. Definition: permission to go onto another's land b. Ex: O allows electrician to come onto O's land to fix an outlet c. A writing isn't required. An invalid oral easement, note, is a license d. Termination: usually revocable at will. May be irrevocable if coupled with an interest or if licensor estopped by licensee's expenditures

D. Chain of title issues 1. To give record notice to later takers, a deed must be recorded properly within the property's chain of title a. The COT is the sequence of recorded docs capable of giving record notice 2. The Shelter Rule a. Anyone who takes from a BFP will prevail against any interest the BFP would've prevailed over. This is true even if the grantee had actual notice of a prior unrecorded conveyance 3. The problem of the wild deed a. Wild deed = a recorded deed that isn't connected to the COT. It doesn't impart constructive notice bc a subsequent purchaser couldn't find it. 4. Estoppel by deed - one who conveys realty in which he has no interest is estopped from denying the validity of that conveyance if he later acquires the title that he had previously purported to transfer. 5. Deeds recorded late - a deed recorded after the grantor parts with title through a subsequent deed isn't constructive notice in most states 6. Reference to another instrument in a recorded document that IS in the chain of title may impart constructive notice of the instrument referred to - even if it's unrecorded or not itself in the COT

E. Judgment creditors 1. A plaintiff who obtains a money judgment can place a judgment lien on D's real property by filing the judgment in the appropriate county office. Whether judgment creditors are protected by the recording statute against a prior unrecorded conveyance by the D depends on the state statute, but most states do not grant them protection. Unlike mortgage lenders, who are the same as any other BFP, judgment creditors and lienors aren't usually protected by the recording acts. Because they aren't covered by the recording act, their notice or lack of notice of prior claims is irrelevant. 2. Most states have statutes requiring a lien holder to record or file the lien in order for it to be enforceable. Don't confuse this with the recording act; this type of statute doesn't mean a lien is protected by the recording act.

Airspace rights: The right to airspace above a parcel is not exclusive, but the owner is entitled to freedom from excessive noise.

E. Right to exclude - remedies of a possessor 1. The possessor of real property has the right to exclude others. Their remedies for invasions include actions for: a. Trespass (land invaded by tangible physical object); b. Private nuisance (land invaded by intangibles such as odors or noise); c. Continuing trespass (land repeatedly invaded by trespasser); and d. Ejectment or unlawful detainer to remove a trespasser or tenant. This action can be joined with a demand for money damages.

An easement is a grant of a nonpossessory property interest that entitles its holder to some form of use or enjoyment of another's land. An easement holder has the right to use another's land for a specified purpose, but that's it. an easement is presumed to be of perpetual duration unless the grant specifically limits it. An affirmative easement is a right to do something on servient land. This includes easements appurtenant and easements in gross.

Easement appurtenant: An easement is appurtenant when it benefits its holder in his physical use or enjoyment of his own land. Two parcels must be involved: 1) A dominant tenement, which derives the benefit 2) A servient tenement, which bears the burden An easement appurtenant is transferred automatically with dominant tenement. The burden of the easement appurtenant also passes automatically with the servient estate, unless the new owner is a BFP without notice of the easement.

Overview of equitable servitudes:

Equitable servitudes are covenants (aka a promise to do or not do something to land) that, regardless of wehther it runs with the land at law, can be enforced in equity against assignees of the burdened land. They are implied from a common scheme for development if notice exists. Requirements for burden to run: intent, notice, touch, and concern Requirements for benefit to run: intent, notice, touch and concern Equitable defenses apply (ie unclean hands, estoppel, acquiescence, changed neighborhood conditions) Generally, equitable servitudes are created by covenants contained in a writing that satisfies the SOF. However, in the absence of a writing, reciprocal negative servitudes may be implied if: 1. There is a common scheme for development (i.e., a plan existing at the time sales of the subdivision parcels began that all parcels be developed within the terms of the negative covenant); and 2. The grantee had actual, record, or inquiry notice of the covenant; and 3. The covenant touches and concerns the land (that is, it benefits the covenantor and the covenantor's successor in their use and enjoyment of the burdened land). The requisite notice may be acquired through actual notice (direct knowledge of the covenants in the prior deeds); inquiry notice (the neighborhood appears to conform to common restrictions); or record notice (if the prior deeds are in the grantee's chain of title the grantee will, under the recording acts, have constructive notice of their contents). Thus, there the restriction need not be in the buyer's record chain of title for the buyer to be burdened by it-as long as the buyer has some kind of notice.

Life tenant must not commit waste. Can't do anything that injures the interests of a remainderman or holder of the reversion. A future interest holder may sue for ds or to enjoy such acts. If they spend money to perform the life tenant's obligations, they're entitled to reimbursement. Exploitation of land's natural resources could constitute voluntary waste.

Exploitation of natural resources by a life tenant is generally limited to situations when: 1. Necessary for repair or maintenance of land 2. The land is suitable only for such use 3. It's expressly or impliedly permitted by the grantor Under open mines doctrine, if mining was done on the land prior to the LE, the life tenant can continue mining, but they're limited to the mines already open

Fee simple subject to a condition subsequent: An estate in which grantor reserves the right to terminate the estate upon the happening of a stated event. Termination isn't automatic - grantor must exercise their right to reentry. Must include an express right of entry for the grantor. If it doesn't include this express language, you've only created a FSD. Most cts hold that rights of entry are not transferable inter vivos, but are devisable by will and descendible through intestacy A conveyance that contains both durational language AND a power of termination will likely be construed as creating a fee simple subject to a condition subsequent and then passes to a third party rather than reverting to the grantor or giving the grantor a right to terminate - third party has an executory interest. In this case, it's like the FSD, except the estate is automatically forfeited in favor of someone other than the grantor.

Exs: "To A and his heirs, but if..." "upon the condition that..." "provided that..." Duration: until happening of named event AND reentry by grantor Correlative future interest in grantor: right of entry Correlative future interest in third party: see fee simple subject to an executory interest

XV. Zoning A. Cities and counties can exercise zoning power only if so authorized by a state enabling act. States can pass zoning/laws for land use based on their general police power. B. The variance is the principal means to achieve flexibility in zoning. 1. The variance grants a landowner permission to depart from the restrictions of a zoning ordinance. 2. What must a proponent of a variance show: The variance is granted or denied by administrative action, typically in the form of a zoning board. Usually, a variance will not be granted if the hardship has been created by the applicant themselves. C. NOTE: Zoning ordinances are generally invalid if they have no reasonable relation to public welfare, are too restrictive, are discriminatory as to a particular parcel, are beyond the grant of authority, violate due process, or are racially discriminatory. D. Nonconforming use 1. If a once lawful, existing use is now deemed nonconforming by a new zoning ordinance, the use can't be eliminated all at once 2. A nonconforming use can't be intensified in ways that constitute a substantial change, but insubstantial changes are permitted. Owners can make reasonable changes. If there is any doubt as to whether a change is substantial or insubstantial, the doubt is resolved against allowing the change.

F. A special use permit is one that must be obtained even though the zoning is proper for the intended use. 1. It is often required for hospitals, funeral homes, drive-in businesses, and so on.

E. Fee simple subject to an executory interest 1. Two exs: a. Fee simple determinable - "To A and his heirs for so long as... and if not... to B." b. Fee simple subject to a condition subsequent: "To A and heirs, but if...to B." 2. Duration: a. In the case of fee simple determinable - as long as condition is met, then to the third party b. In the case of fee simple subject to a condition subsequent - until the happening of the event 3. Correlative future interest in a third party: executory interest

F. Fee tail 1. Most js have abolished. Attempt to create fee tail results in fee simple 2. Exs: "TO A and the heirs of his body" 3. Duration: til A and his line die out 4. Correlative future interest in grantor: reversion 5. Correlative future interest in third party: none (but remainder is possible)

6. Senior Interests a. Foreclosure does not affect any interest senior to the mortgage being foreclosed. The buyer at the sale takes subject to such interest. b. Is the buyer personally liable on the senior debt? No. But as a practical matter, if the senior mortgage is not paid, sooner or later the senior creditor will foreclose against the land. 7. Priorities a. As a CR, you must record. Until you record, you have no priority. Once recorded, priority is determined by first in time, first in right b. Note though: by private agreement, a senior CR may agree to subordinate its priority to a junior CR. These are acceptable 8. Redemption a. Redemption in Equity 1) Equitable redemption is universally recognized up to the date of sale. What that means is that at any time prior to the foreclosure sale the debtor has the right to redeem the land by freeing it of the mortgage. 2) Once a valid foreclosure has taken place: right to equitable redemption is cut off.

F. Mortgage alternatives 1. Deed of Trust a. Some states call a security interest in land a deed of trust rather than a mortgage. The debtor/notemaker is the trustor. The trustor gives a deed of trust to a third-party trustee, who is usually closely connected to the lender (the beneficiary). On default, the lender instructs the trustee to foreclose the deed of trust by sale. 2. Absolute Deed a. An absolute deed, if given for security purposes, can be treated as an "equitable" mortgage (CR must still foreclose by judicial action). 3. Installment Land Contract a. An installment purchaser obtains legal title only when the full k price has been paid off. Forfeiture clauses, allowing the vendor upon default to cancel the contract, retake possession, and retain all money paid, are common and generally enforceable. 4. Equitable Vendor's Lien a. This lien does not result from an agreement; arises by implication of law when a seller transfers title to the buyer, and the purchase price or a portion of it remains unpaid. 5. Sale - leaseback a. A landowner may sell her property for cash and then lease it back from the purchaser for a long period of time. Like an absolute deed, this may be treated as a disguised mortgage.

V. The rules against restrains on alienation A. Generally, any restriction on the transferability of a legal interest is void B. There are three types of restraints on alienation: 1. Disabling restraints, under which attempted transfers are ineffective; 2. forfeiture restraints, under which an attempted transfer forfeits the interest; and 3. promissory restraints, under which an attempted transfer breaches a covenant. C. A disabling restraint on any legal interest is void. Forfeiture and promissory restraints may be valid, depending on the nature of the restraint and the interest involved. D. Restraints on a fee simple 1. All absolute restraints on fee simple estates are void; thus, any such restriction falls away and the grantee may freely transfer the property. A partial restraint is one that purports to restrict the power to transfer to specific persons, or by a specific method, or until a specific time. Forfeiture or promissory restraints on fee simple estates for a limited time and reasonable purpose may be upheld (ex, this might look like a restraint on transfer that's limited to the joint lifetimes of co-owners, which could be seen as a reasonable way to ensure that neither co-owner would have to reside with a stranger). E. Discriminatory restraints 1. Judicial enforcement of restraints prohibiting the transfer or use of property to or by a person of a specified racial, religious, or ethnic group is discriminatory state action forbidden by 14th Amendment. Discriminatory restrictions may also violate the FHA.

F. Restraints on a life estate 1. Forfeiture and promissory restraints on life estates are valid, but disabling restraints are void. G. Remember that the Rule Against Restraints on Alienation applies only to legal interests. Restraints on the alienation of equitable interests (for ex, spendthrift clauses in trust instruments) are valid. H. Restraints on future interest 1. Restraints on vested future interests generally are valid to the extent that restraints on present interests of the same type are valid. For example, forfeiture and promissory restraints on vested remainders for life are valid, but disabling restraints on vested remainders for life are void. I. Other valid restraints on alienation: 1. Reasonable restrictions in commercial transactions 2. Reasonable options and rights of first refusal 3. Restrictions on assignment and sublease of leaseholds (ex, requiring landlord's consent)

Land sale ks are typically subject to the SOF. The exception to this is partial performance - namely, no writing is required for a real estate k to be enforceable if the buyer has performed 2 of these 3: possession, improvement, or payment. Under a land sale k, time is not presumed to be of the essence. Land sale ks contain an implied covenant that seller will deliver title free from an unreasonable risk of litigation at closing (i.e., when purchase price and deed exchanged).

For a deed to be valid, it must 1) evidence an intent to transfer land and 2) adequately describe the land and 3) the parties. A deed is effective on delivery and acceptance. Types of deeds 1) General warranty deed—covenants against any title defects created by the grantor or prior titleholders 2) Special warranty deed—covenants against title defects created by the grantor 3) Quitclaim deed—no covenants; transfers whatever interest grantor has

A deed isn't effective to transfer an interest unless it's been delivered. Delivery is based on grantor's intent that title pass immediately, even if physical possession of the deed is postponed. Acceptance by the grantee is presumed. Delivery can be satisfied by physically delivering a deed to the grantee. However, if grantee expressly reject the deed, deed is ineffective to pass title. All types of extrinsic evidence is admissible to prove the grantor's intent to transfer title. But outside evidence not permitted to show that an unconditional deed given directly to a grantee was subject to an oral condition. A delivery to a third party with instructions to deliver the deed to the grantee is considered valid delivery. Whether a delivery to a third party without instructions is a valid delivery often hinges on whether that third party is an agent of the grantor or the grantee. Delivery to the grantor's lawyer is probably not delivery, while delivery to the grantee's lawyer probably is.

Grantor may deliver an executed deed to a third party/escrow agent, with instructions that the deed be delivered to the grantee once certain conditions are met. Once the conditions are met, if the escrow agent is given written instructions, the grantor is bound by the delivery to the agent. But, if the grantor gives the escrow agent oral instructions, the grantor may change the instruction and recall the deed while it's still in the agent's hands, unless there's a written contract of sale.

E. Lives in being requirement 1. You are looking for a person alive at the date of the conveyance whose life and/or death is relevant to the condition's occurrence. 2. Determine if we'll know for sure within 21 years of the measuring life's death if the future interest holders can take. If so, the conveyance is good. If we aren't sure if we'll know positively within 21 years of the measuring life's death whether the future interest holder can take, the future interest is void. F. Interests exempt from the rule 1. Except for vested remainders subject to open, RAP doesn't apply to vested interests. Thus, other vested remainders, reversions, possibilities of reverter, and rights of entry are not subject to the Rule. Moreover, there is a charity-to-charity exception to the Rule (that is, the Rule does not apply to any disposition from one charity to another), and an exception for options to purchase held by a current tenant. G. Violation of the RAP only destroys the offending interest 1. But note: When a void interest is stricken, the interests are classified as if the void interest were never there. Ex, if O conveys "to A for as long as no liquor is consumed on the premises, then to B," B's interest would be stricken, A would have a fee simple determinable, and O would have a possibility of reverter. In contrast, if O conveys "to A, but if liquor is ever consumed on the premises, then to B," B's interest and the condition are stricken, and A has a fee simple absolute. That's because now, once the offensive future interest is stricken ("then to B"), the conveyance is no longer grammatically sound. Hence, the entire language of condition is stricken, leaving us with "to A." A has a fee simple absolute.

H. RAP in operation - common pitfall cases 1. Bright line rule - an executory interest with no time limit violates the rule a. An executory interest that follows a defeasible fee, with no limit on the time within which it must vest (for example, "to A for so long as no liquor is consumed on the premises, then to B"), violates RAP, and the executory interest is stricken. (An executory interest following a defeasible fee is valid only if the condition is specific to the fee holder or expressly limited to the perpetuities period.) 2. A gift to an open class that is conditioned on its members surviving beyond age 21 violates the rule a. Ex: "To A for life, then to those of A's children who attain the age of 25." A is alive. She has one child, B, who is 30. The remainder in A's children violates the Rule and is void, bc A's still alive. Thus, the class is still open. B could die tomorrow. A could die thereafter, without having had another child, or A could live, have another child, and die in labor or live. We won't know today, at the time of the grant's creation, whether A will have a child to reach 25 within 21 years of A's death. Thus, the gift is void b. Note that if the interest of any class member may vest too remotely, the whole class gift fails. For the class gift to vest, the class must be closed and all conditions precedent must be satisfied for every member 3. Fertile octogenarian a. Woman is presumed to be capable of bearing kids so long as she's alive. Note that a few states have statutes that raise a presumption a woman can't have kids over a certain age.

4. Unborn widow a. Bc a person's widow isn't determined til their death, it may turn out to be someone e who wasn't in being at the time of the disposition. b. Exs: 1) O conveys to A for life, then to A's widow for life, then to A's surviving issue in fee." In the absence of a statute to the contrary, the gift to A's issue is invalid because A's widow might be a spouse who wasn't in being when the interest was created. 2) Compare: a remainder "to A's kids" is valid they'd be determined at A's death. c. A few states do have statutes that raise a presumption that any reference to a person's widow is in being at the time of transfer 5. A gift conditioned on an administrative contingency (ex, admission of will to probate) violates the Rule a. Ex: a gift "to my issue surviving at the distribution of my estate" is invalid bc the estate might be administered beyond the period of the rule. b. Some states raise a presumption that the transferor intended the contingency should occur within 21 years

I. Application of RAP to class gifts 1. Each gift to a subclass may be treated as a separate gift under the rule a. Ex: "Income to A for life, then to A's children for their lives. Upon the death of each of A's children, the corpus is to be distributed to that child's issue, per stirpes." The gifts to each of A's children's issue are considered separately. Thus, the gifts to issue of A's children living at the time of the disposition are good, but the gifts to the issue of afterborn children of A violate the Rule and are void. g. Per capita gift exception: a gift of a fixed amount to each member of a class isn't treated as a class gift under the rule 1) Ex: "$1,000 to each of my grandkids, whether born before or after my death." This creates gifts to individuals, each of whom is judged separately under the Rule.

D. Transferability of remainders and executory interests 1. Vested remainders are fully transferable, descendible by intestacy, and devisable by will. At common law, contingent remainders and executory interests were not transferable inter vivos, but most courts today hold that they are freely transferable. Contingent remainders and executory interests are descendible and devisable, provided survival is not a condition to the interest's taking. 2. NOTE: Any future interest that is transferable is subject to involuntary transfer; that is, it is reachable by creditors.

IV. The Rule against perpetuities (RAP) A. RAP provides that certain kinds of future interests are void if there is any possibility at all that the interest might vest more than 21 years after a person who was alive at the time of the grant has died. C. The time the perpetuities period begins to run depends on the instrument and interest created 1. For interest granted by will, period runs from the date of the testator's death 2. For deeds, it's the date of delivery 3. Period runs on an irrevocable trust from the date it's created 4. Runs on a revocable trust from the date it becomes irrevocable D. Vesting rule 1. An interest vests for RAP purposes when it becomes possessory, or an defeasible vested remainder, or a vested remainder subject to total divestment

When there are competing interests in land, the recording acts protect a bona fide purchaser for value without actual, inquiry, or record notice of the prior conveyance ("BFP"). a. Actual notice - what the grantee actually knows b. Inquiry notice - what a reasonable inquiry would've revealed c. Record notice - what a search of the real property records would've revealed Types of recording acts: a. Notice statutes - later BFP wins if earlier grant wasn't recorded b. Race-notice statutes - later BFP wins only if she records before the earlier grantee records c. Race statutes - first to record wins; actual notice is irrelevant

If mortgagor transfers mortgaged land, the grantee may agree to assume the mortgage and become primarily liable to pay the mortgage loan. A grantee who doesn't assume the mortgage isn't personally liable for the loan, but he may lose the land if transferor defaults Foreclosure - after default, property may be sold to satisfy the debt. Foreclosure doesn't affect any interests that are senior to the interest being foreclosed on. However, foreclosure terminates any interests that are junior to the foreclosed interests. Junior interests are entitled to any surplus remaining after the foreclosing mortgage is satisfied. Mortgagor may redeem the land by paying the amount due Deficiency - mortgagee can sue mortgagor if foreclosure sale proceeds don't satisfy mortgage debt

Present possessory interests are divided into defeasible and indefeasible interests:

Indefeasible interests - not subject to early termination (fsa and life estate) Defeasible interests - allows a fee simple or life estate to be terminated if a stated event occurs (fsd, subject to condition subsequent, and subject to an executory interest)

Future possessory interests include:

Interests retained by the grantor: 1. reversion 2. possibility of reverter 3. right of entry Interests created in a transferee: 4. executory interests 5. remainders, 6. class gifts

What is a tenancy in common:

It is a form of concurrent co-ownership. Each tenant has a distinct, proportionate, undivided interest in the property. There is no right of survivorship. Each interest is devisable, descendible, and alienable. Each co-tenant owns an individual part, and each has a right to possess the whole. Ex: "To A and B," or, sometimes, "To A and B as joint tenants." The only unity required to create a TIC is possession. TIC may be terminated by partition. If we have a conveyance with multiple grantees, we presume that they'll take as TIC, not JTs.

Title must be marketable on the day of closing. Seller has up until that time to clear up whatever defect is making it unmarketable. Note: In an installment land k, seller doesn't have to provide marketable title til the buyer makes his last payment. Once closing occurs, seller is no longer liable for this implied contractual covenant. Seller only liable for promises made in the deed. If title isn't marketable, buyer must notify the seller and give the seller reasonable time to cure the defects. If seller doesn't cure, buyer's remedies include: Rescission, ds, specific performance with abatement, quiet title suit Note that a k calling for a quitclaim doesn't in any way affect the implied covenant to provide marketable title.

Land sale k contains no implied warranties of fitness or habitability. Caveat emptor is the common law norm. Exception: most cts recognize a warranty of fitness in the sale of a new home by the builder. In this instance, a person may sue a builder for negligence in performing a building k.

Lapse and anti-lapse statutes: A lapse occurs when the beneficiary of a gift in a will dies before the testator. Under the common law, this gift was void. However, nearly all states now prevent lapse by permitting the gift to pass to the predeceasing beneficiary's living descendants under certain circumstances. These statutes vary as to the scope of beneficiaries covered by the statute.

Many anti-lapse statutes apply only when the named beneficiary is a descendant of the testator. Others apply if the beneficiary is more remotely related. Others apply to any relative, and still others apply to any beneficiary at all. Application to Class Gifts: If a class member within the coverage of an anti-lapse statute predeceases the testator leaving surviving issue, the statute will apply and the issue will take the deceased class member's share of the gift. The anti-lapse statute does not apply if there is a contrary will provision; for example, the gift is contingent on the beneficiary's surviving the testator.

If the buyer of land determines that the seller's title is unmarketable, the buyer must notify the seller and give a reasonable time to cure the defects. Every land sale contract contains an implied covenant that the seller will provide marketable title at closing.

Marketable title is title reasonably free from doubt, which a reasonably prudent buyer would accept. While it need not be perfect title, it must not present the buyer with an unreasonable risk of litigation. Generally, this means an unencumbered fee simple with good record title.

Overview of real covenants (that run with the property):

Real covenants are written promises to do or refrain from doing something on land, with a usual remedy of money ds Requirements for burden to run to later grantees; intent, notice, horizontal privity, vertical privity, touch and concern Requirements for benefit to run: intent, vertical privity, touch and concern

When there is concurrent ownership, each co-tenant has the right to possess all portions of the property; they have no right to exclusive possession of any part. If one co-tenant wrongfully excludes another co-tenant from possession of the whole or any part, they've committed ouster. A repairing co-tenant enjoys a right to contribution for reasonable, necessary repairs from their co-tenants, provided they give the co-tenants notice of the need for those repairs. However, if one tenant just performs "improvements," they have no right to demand contribution from the other co-tenants. There is a duty of fair dealing among co-tenants.

Special situations to know when a co-tenant is in exclusive possession of the property: 1. If one of the co-tenants leases all or part of the premises to a third party, they must provide their co-tenants with their fair share of the rental income. 2. However, a co-tenant in exclusive possession of the property has the right to retain profits from their use of the property - they don't have to share the profits with other co-tenants, absent ouster or an agreement to the contrary. 3. Unless they've committed ouster, a co-tenant in exclusive possession can't acquire title to the whole to the exclusion of the other co-tenants.

How might a joint tenancy with a right of survivorship be terminated:

The right of survivorship may be severed, and the estate converted to a TIC, by: 1. A conveyance by one joint tenant 2. Agreement of joint tenants 3. Murder of one co-tenant by another 4. Simultaneous deaths of the co-tenants 5. By petition (voluntary agreement, a forced sale, or a petition in kind. A petition in kind is a judicial action for a physical division of the property, if in the best interests of all parties. a Forced sale is a judicial action where the land is sold and the sale proceeds are divided up proportionately, when doing so is in the best interest of all parties). Note that in a lien theory states, mortgages will not cause a severance. It will create a severance, however, in title theory states.

Running of the statute for adverse possession: begins to run when the true owner can first bring suit. Filing a suit won't stop the period from running, though - suit must be pursued to judgment Most states don't require the adverse possessor to pay taxes on the property, but consider such payment good evidence of a claim of right Tacking: one adverse possessor may tack onto his time with the land his predecessor's time, so long as there is privity between the possessors. Privity is satisfied by any non-hostile nexus like k, deed, or will. Privity is absent when the possessor acquires possession by ousting his predecessor in possession.

The statute of limitations won't run against a true owner who is afflicted with a disability at the inception of the adverse possession. The statue of limitations doesn't run against the holder of a future interest until their interest becomes possessory. The event or condition giving rise to a grantor's right of entry (ex "To Grantee on condition that if alcohol is ever used on the premises, Grantor shall have the right to reenter and retake the premises") doesn't trigger the statute of limitations for purposes of adverse possession. Statute doesn't start to run til the grantor asserts their right. If an adverse possessor uses the land in violation of a restrictive covenant in the owner's deed for the statute of limitations period, they take free of the restriction. If, however, the possessor's use complies with such a covenant, they take title subject to the restriction. Title to government-owned land and land registered under a Torrens system (whereby title is est with a governmental authority that issues title certificates to owners) can't be acquired by adverse possession.

What is a tenancy by the entirety:

This is a form of concurrent ownership that only exists between spouses. Each spouse has an undivided interest in the whole estate and a right of survivorship Ex: "To A and B" (when A and B are married). Note that some states presume a Tenancy by the entirety in any joint conveyance to two spouses where the four unities are present. This type of ownership may be severed by: 1) Death, 2) Divorce (converts to a TIC) 3) Mutual agreement, or 4) Execution by a joint creditor It CANNOT be terminated by involuntary petition. Creditors of only one spouse can't touch the tenancy for satisfaction of the doubt. Furthermore, one spouse acting alone can't defeat the right of survivorship by unilaterally conveying to a third party, and just one spouse can't encumber the land - a deed or mortgage just executed by one spouse is ineffective

What is a periodic tenancy:

This is a tenancy for some fixed period that continues for succeeding periods until either party gives notice of termination. Ex: "To T from month to month," OR "To T, with rent payable on the first day of every month," OR L elects to bind hold-over T for an additional term. It can also arise by implication in these three situations: 1) Land is leased with no mention of duration, but provision is made for payment at set intervals 2) An oral term of years in violation of SOF creates an implied periodic tenancy, measured by the way rent is tendered. 3) LL holds over a T who has wrongfully stayed on past the og lease's end. Implied tenancy arises, as measured by the way rent is tendered This tenancy ends by notice from one party that's at least equal to the length of the time period. Eg, one full month for a month to month tenancy. The exception to this: only one month's notice (six months' at common law) is required to terminate a year to year tenancy

What is a tenancy at will:

This is a tenancy of no stated duration that lasts as long as both parties desire. Ex: "To T for and during the pleasure of L." With this type of tenancy, the LL or tenant either one may terminate at any time. Unless parties expressly agree to a tenancy at will, the payment of regular rent will cause the ct to imply a periodic tenancy. Note: If the lease gives only the ll the right to terminate, a similar right will be implied for the tenant. But if only the tenant has a right to terminate, a similar right won't be implied in favor of the ll. This type of tenancy usually ends once one party displays an intention that the tenancy should come to an end. It may also end by operation of law (eg, death of a party, attempt to transfer interest).

What is a tenancy for years:

This is a tenancy that lasts for some fixed period of time. Ex: "To T for x years." This type of tenancy terminates at the end of the stated period, without either party giving notice. It also terminates if tenant breaches any of the lease's covenants, like failure to pay rent. This will end the lease even if the LL didn't include in the lease a right of entry for themselves. Termination also occurs if the tenancy surrenders the tenancy and the LL accepts. However, if the unexpired term exceeds one year, the surrender must be in writing.

What is a joint tenancy with a right of survivorship?

This is a type of concurrent ownership. Each tenant has an undivided interest in the whole estate, and the surviving co-tenant has a right to the whole estate (right of survivorship). Ex: "To A and B as joint tenant with the right of survivorship." Must have this survivorship language, for without it, it may be construed as a TIC. To create a joint tenancy, the joint tenants must take (the four unities): 1) Identical interests 2) From the same instrument 3) At the same time 4) With an equal right to possess A joint tenant's interest is alienable inter vivos. A joint tenant's interest is neither devisable nor descendible. A deceased joint tenant's property passes to the surviving joint tenant by operation of law.

What is a tenancy at sufferance:

This tenancy occurs when a Tenant wrongfully holds over after termination of the tenancy. That is, T's lease expires, but T continues to occupy the premises. If the rent was computed on a monthly basis, the election creates a month-to-month periodic tenancy. If rent was computed on a quarterly basis though payable monthly, a quarter periodic tenancy is created, and so on. No matter how the rent is computed, the maximum tenancy that can be created by the election to hold the tenant to another term is a year-to-year tenancy. The various promises made by the landlord and the tenant in the original lease become part of the tenancy for the additional term. Exceptions - a LL can't bind T to new tenancy when: 1) The tenant remains in possession for only a few hours after termination or leaves a few articles of personal property or 2) the delay is not the tenant's fault (example, severe illness). This type of tenancy ends when the L evicts T or elects to hold them to another term.

The Riparian Doctrine:

Under this doctrine, the water belongs to those who own the land bordering the watercourse. Riparian rights attach to all contiguous tracts held by the same owner as long as one of the tracts abuts the water. Riparian owners can use water only in connection with the riparian parcel. 1) Natural Flow Theory - Riparian owner's use is enjoinable if it results in substantial or material diminution of the water's quantity, quality, or velocity. 2) Reasonable Use Theory - All riparians share the right of "reasonable use" of the water (that is, one owner's use is not enjoinable unless it substantially interferes with the use of other riparian owners). In determining "reasonable" use, courts balance the utility of the owner's use against the gravity of the harm. Six factors used to make this determination: alteration of flow, purpose of use, pollution, extent of use, destination of water taken, and miscellaneous conduct that may give rise to litigation. Under either theory, natural uses (human uses, ex, consumption, gardening) prevail over artificial uses (ex, irrigation, manufacturing).

Damages and remote grantees: Present covenants can't be enforced by remote grantees, but future covenants run with the grantee's estates. If there are successive conveyances by general warranty deed and the last grantee is evicted by lawful claim of title, he may sue anyone up the line. Some states allow him to recover to the extent of consideration received by a defendant-covenantor. Other states limit recovery to the lesser of what he paid or what the defendant-covenantor received.

Watch for a situation in which a joint owner attempts to convey property by forging the signature(s) of the other owner(s). Such a conveyance would be valid as to the interest of the owner whose signature is genuine but void as to the other owner(s). Thus, if one joint tenant executes a deed for the entire property with his own signature and the forged signature of the other joint tenant, the conveyance works a severance; the buyer would hold as a tenant in common with the joint tenant whose signature was forged.

Restrictive covenants - a covenant is a written promise to do, or not do, something on land. Covenants can be negative or affirmative. Negative/restrictive covenants = a promise to not do something on land. Affirmative covenants = promises to do something related to land. Howe do you distinguish between a covenant and an equitable servitude? Look at the ds the P is requesting. If they want money ds, construe it as a covenant. If they want an injunction, construe it as an equitable servitude. The complicated part of covenants is understanding the running of its benefit and burden.

Way to understand the running of the burden and benefit of a covenant: A promises B that A won't build for commercial promises on A's property. A sells to A-1, B sells to B-1. B-1 wants to prevent A-1 from building commercially. A is the burdened parcel, B is the benefitted parcel. Separate requirements are in place as for whether the burden runs from A to A-1, and whether the benefit runs from B to B-1.

Scope of easements - Generally, scope is determined by the grant or conditions that created the easement. If an easement is created but not specifically located on the servient tenement, an easement of sufficient width, height, and direction for the intended use will be implied. The owner of the servient tenement may select the location of the easement so long as their selection is reasonable. If there are no specific limitations in the grant, courts assume that an easement was intended to meet both present and future needs of the dominant tenement (for ex, an easement may widen to accommodate new, wider cars). If, however, the dominant parcel is subdivided, the lot owners won't succeed to the easement if to do so would unreasonably overburden the servient estate.

When confronted with an exam question involving overuse or misuse of an easement, remember that such use does not terminate the easement. The appropriate remedy for the servient owner is an injunction against the misuse. The servient owner generally may use her land in any way she wishes so long as her conduct does not interfere with the easement. The easement holder has the duty to make repairs to the easement if she is the sole user; but if both the easement holder and the servient owner are users, the court will apportion the repair costs.

XVI. Condos and HOAs A. With a condo, each owner owns the interior of their individual unit, plus an undivided interest in the exterior and common elements. B. HOAs 1. Each condo owner is a HOA member. Members vote to elect a board which manages the property. The association is usually a legal entity such as a corp or LLC 2. Many HOAs pass charter and bylaws that place requirements on each owner's use of the property. These are called a CC&R. 3. Each condo owner must pay regular dues to the HOA.

XVII. Natural Rights, aka rights that are incidental to ownership of land A. In general, an owner of real property has the exclusive right to use and possess the surface, the airspace, and the soil of the property. B. Rights to lateral and subjacent support of land 1. Lateral support a. Ownership of land includes the right to have the land supported in its natural state by adjoining land. b. A landowner is strictly liable if their excavation causes adjacent land to subside (meaning, slip or cave in). c. If land is improved by buildings and an adjacent landowner's excavation causes that improved land to cave in, the excavator will be liable only if negligent. However, strict liability may apply. For strict liability to apply, P must show that the improvements on their land didn't contribute to their land's collapse. b. Subjacent Support 1) An underground occupant of land (ex, a mining co) must support the surface and buildings existing on the date the subjacent estate was created. Liability for subsequently erected buildings requires negligence.

A leasehold is an estate in land in which the tenant has a present possessory interest in the land, and the landlord has a future interest (reversion). There are four leasehold estates:

a. Tenancy for years b. Periodic tenancy c. Tenancy at will d. The tenancy at sufferance

What are the three types of leasehold interests that may be created in property:

a. Tenancy for years - for a fixed period of time 1) Created expressly, ends automatically on its termination date (no notice) b. Periodic tenancy - for a fixed period that continues for succeeding periods (aka, month to month) 1) Created expressly or when a lease draws periodic rent payments, terminated on proper notice (appropriate time period) c. Tenancy at will - no stated duration, as long as parties desire 1) Created expressly, terminated on proper notice d. Tenancy at sufferance (hold-over doctrine) - tenant remains in possession after tenancy expires 1) LL may evict tenant or create a periodic tenancy by accepting rent

3) the vested remainder subject to open a) ex: To A for life, then to A's kids in equal shares b) is transferable, descendible, and devisable c) this is subject to the RAP, as long as the class remains open d) this is a vested remainder created in a class of persons that's certain to become possessory, but it's subject to diminution - for ex, by the birth of additional persons who will share in the remainder as a class.

c. Class gifts 1) A class is a group of persons having a common characteristic. The share of each member is determined by the number of persons in the class. 2) A class gift of a remainder may be vested subject to open (where at least one group member exists) or contingent (where all group members are unascertained) 3) A class is open when it's still possible for others to join. It is closed when no others can join in. 4) To figure out if a given class has closed, we apply the rule of convenience - in the absence of an express contrary intent, a class closes when some member of the class can call for distribution of their share of the class gift. a) Exception: the womb rule - persons in gestation at the time the class closes are included in the class 5) Exs: a) T's will devises property to W for life, then to A's children. At the time the will is executed, A has two children, B and C. A then has another child, D. T dies. A has child E, then W dies. After W's death, A has another child, F. The class closed at W's death because it was time to make the distribution. Thus, B, C, D, and E share the property, and F is excluded. b) T's will devises the residue of his estate "to those of A's children who attain age 21." If any of A's children is 21 at T's death, the class closes at that time. Otherwise it closes when one of A's kids reaches age 21. But remember, if it had been a future gift (that is, "to A for life, then to such of A's children who attain age 21"), the class would remain open until the life tenant's death even if some of the class members had reached the stated age at T's death. 6) Survival of a class member to the time of closing is usually unnecessary to share in a future gift, unless survival was made an express condition (ex, to A for life and then to her surviving kids). However, certain terms are construed to create implied survivorship conditions (ex, widow, issue, heirs, next of kin).

3. Rule Against Perpetuities (RAP) a. Any future interest that's not certain to vest or fail within a life in being plus 21 years is void b. Applies to contingent remainders, executory interests, class gifts (even if vested remainders), options and rights of first refusal, and powers of appointment c. Doesn't apply to vested interests, grantors' reversionary interests, or gifts between charities d. Only the interest that violates the Rule is stricken (severed from the disposition) e. Cases that always violate the common law Rule: 1) Executory interest following a defeasible fee—executory interest is stricken 2) Gift to an open class conditioned on members surviving to an age beyond 21—entire class gift is stricken ("bad as to one, bad as to all") 3) Remainder to A's children living at his widow's death ("unborn widow" problem)—contingent remainder is stricken 4) Gift conditioned on an administrative contingency is stricken 5) Options or rights of first refusal that extend to the holder's heirs and assigns are stricken

f. At common law, a woman is conclusively presumed capable of bearing children (the "fertile octogenarian") g. Departures from common law 1) "Wait and see" statutes—validity of interest determined by actual future events 2) Uniform Statutory Rule Against Perpetuities—90-year vesting period, "wait and see" approach 3) Cy pres approach—invalid interests reformed to match grantor's intent • Future and present interests • Devisable: capable of passing by will • Descendible: capable of passing by intestacy • Alienable: capable of transfer inter vivos (during lifetime) • FSA: "To A" or "to A and his heirs." Potentially limitless duration. Devisable, descendible, alienable. • Defeasible fees: these are fee simples, with a condition attached (defeasance = forfeiture) • First of defeasbiel family: FSD • Fee simple determinable: terminates automatically on the happening of a stated event. Created by clear durational language: so long as; while; during; until. Devisable, descendible, alienable.

There are three types of concurrent ownership:

joint tenancy (JT), Tenancy by the entirety, tenancy in common (TIC)

• If four years into a six-year tenancy for years, T orally transfers his entire interest to T2 for two years, the attempted assignment is ineffective under the SOF. • If one year into a five-year tenancy for years, T transfers his interest "to T2 for four years; however, if T2 breaches the original lease terms, T may reenter and retake the premises," this is an assignment of the lease from T to T2. A complete transfer of the entire remaining lease term constitutes an assignment. If tenant retains any part of the remaining lease term, OTHER than a right of reentry for breach of the og lease terms, the transfer is a sublease. • An assignee is in privity of estate with the landlord and is liable for all covenants that run with the land, including the covenant to pay rent. The original tenant (assignor) remains in privity of contract with the landlord and is liable for the rent reserved in the lease if the assignee abandons the property.

• A landlord's promise in a lease to maintain the property does not terminate bc the property is sold. The og LL and tenant remain in privity of k, and the og LL remains liable on the covenant unless there is a novation. A novation substitutes a new party for an og party to the k. It requires the assent of all parties and completely releases the og party. When leased property is sold, the purchaser may be liable for his predecessor's promises if the promise runs with the land. A covenant in a lease runs with the land if the parties to the lease so intend and the covenant touches and concerns the land. Generally, promises to do a physical act, such as maintain or repair the property, are considered to run with the land. If an easement is said to be surcharged, this means the easement's legal scope was exceeded. The holder of an easement has the right to use another's land (i.e., the servient tenement), but has no right to possess the land. The scope of an easement is determined by the reasonable intent of the original parties, and when the scope has been specified, these specifics will govern. However, when an easement's scope has been set out only in general language, cts will interpret it to accommodate the holder's present and future reasonable needs. In either event, if the easement holder uses the easement in a way that exceeds its legal scope, the easement is surcharged. The servient landowner may enjoin the excess use and possibly sue for damages if the land has been harmed.

Mortgages, easements, restrictive covenants will render a title unmarketable, UNLESS the buyer has expressly waived them (which most buyers do). • What if a deed that's absolute on its face says "the land is yours only if you survive me:" the condition falls out, the grantee takes without being subject to the term. • Six covenants for title in a general warranty deed: - Present covenants (breached at delivery): a. Seisin: the grantor owns this estate b. Right to convey: grantor can transfer c. Against encumbrances: no servitudes/liens - Future covenants (breached if grantee disturbed in possession) a. Quiet enjoyment: no third party lawful claims b. Warranty: grantor will defend grantee should there be any reasonable claim of title asserted by others. c. Further assurances: grantor will perfect. If it turns out something is wrong, promises to do whatever is needed later to perfect the title. More of a housekeeping assurance. • Estoppel by deed: - Grantor purports to convey to grantee realty they don't then own - Grantor later acquires title to the property - Title automatically vests in the grantee - Grantor is estopped from denying validity of preacquisition conveyance - But watch out for BFP: early recording is outside the chain of title

• A mortgage must be in writing to satisfy the SOF. • PMSI for land: has first priority in the financed parcel. • DO cannot waive their right to redeem in the mortgage itself - this is what we call "No clogging the equity of redemption." • Zoning: nonconforming use - previously allowed use can't be eliminated all at once unless just compensation is paid • Cumulative ordinance: a higher use type of building is allowed into a land that's zoned for a lower use purpose. But NOT vice versa. • In a HOA: if there's a special expense, there'll be a special assessment = one time fee if dues don't cover an expense • If L leases property to T, and L subsequently assigns L's interest to L2, T may hold L or L2 liable when X, a paramount title holder, ejects T. A landlord may assign the rents and reversion interest that he owns. The assignee is liable to the tenants for performance of all covenants made by the original landlord in the lease, provided that those covenants run with the land. The og landlord also remains liable on all of the covenants he made in the lease. • If a tenant transfers her interest in violation of a covenant against assignment or sublease, the transfer is NOT void. However, the landlord usually may terminate the lease under the lease terms or a statute or sue for damages.

The benefit of an equitable servitude runs to successors if: (i) the original parties so intended, and (ii) the servitude touches and concerns the land. The burden runs if (i) and (ii) are met and (iii) the subsequent purchaser has actual or constructive notice of the covenant. Privity of estate is not needed to enforce an equitable servitude because it is enforced not as an in personam right against the owner of the servient tenement, but as an equitable property interest in the land itself. • An existing violation of a zoning ordinance would render title to land unmarketable. Every land sale contract contains an implied covenant that the seller will provide marketable title at closing. Marketable title is title reasonably free from doubt, which a reasonably prudent buyer would accept. While it need not be perfect title, it must not present the buyer with an unreasonable risk of litigation. Generally, this means an unencumbered fee simple with good record title. The mere existence of a zoning ordinance does not constitute an encumbrance. However, title to land that currently violates a zoning ordinance would be considered unmarketable.

• A mortgage that the seller is poised to satisfy at closing would not render title to land unmarketable. A seller has the right to satisfy a mortgage or lien at the closing with the proceeds from the sale. Thus, as long as the purchase price is sufficient and this is accomplished simultaneously with the transfer of title (e.g., through the use of escrows), the buyer cannot claim that title is unmarketable; the closing will result in a marketable title. • A visible easement that benefits the property would not render title to land unmarketable. Most courts hold that a beneficial easement (e.g., a utility easement) or one that was visible or known to the buyer does not constitute an encumbrance. In contrast, an easement that reduces the value of the property or is unknown to the buyer constitutes an encumbrance that renders title unmarketable. • A very slight encroachment onto an adjacent landowner's land would not render title to land unmarketable. An encroachment will not render title unmarketable if: 1. It is very slight (only a few inches) and does not inconvenience the owner on whose land it encroaches; 2. The owner encroached upon has indicated that he will not sue on it; or 3. It has existed for so long (many decades) that it has become legal by adverse possession (if the state recognizes adverse possession title as marketable). In contrast, a significant encroachment constitutes a title defect that renders title unmarketable.

• Condemnation of the servient estate will terminate an easement. The easement holder may be entitled to compensation for the value lost. Use of the easement beyond its legal scope will not terminate an easement. Instead, the easement is surcharged, and the servient owner may sue to enjoin the use. Nonuse of the easement for the statutory period will not terminate an easement. An easement can be extinguished by the easement holder's physical act of abandonment (e.g., erection of a permanent structure over the easement). However, mere nonuse, even for a long period of time, is insufficient to constitute an abandonment of the easement. To terminate the easement, the nonuse must be combined with other evidence of intent to abandon it. Voluntary destruction of the servient estate (e.g., tearing down a building to erect a new one) will not terminate an easement. But involuntary destruction of the servient estate (e.g., by fire or flood) will extinguish the easement. An easement MAY be terminated by prescription. However, this is not accomplished through the easement holder's nonuse. Rather, one must interfere with the easement through long continued possession and enjoyment of the servient estate in a way that would indicate to the public that no easement right existed. This is analogous to a landowner losing title to land by adverse possession. Generally, the release of an easement interest must satisfy the SOF. However, a release is only one way to terminate an easement. A release from the owner of the easement interest to the owner of the servient tenement is effectively a conveyance and must satisfy the formalities required to create an easement.

• An easement may be implied if, prior to the time the tract is divided, a use exists on the "servient part" that is reasonably necessary for the enjoyment of the "dominant part," and parties intended the use to continue after division of the property. To give rise to an easement, a use must be apparent and continuous at the time the tract is divided. The use must also be reasonably necessary. Whether a use is reasonably necessary depends on many factors, including the cost and difficulty of the alternatives. The holder of the servient estate, has the right to choose the location of an easement by necessity. An easement by necessity arises when the owner of a tract of land sells a part of the tract and by this division deprives one lot of access to a public road or utility line. The owner of the servient parcel has the right to locate the easement, provided the location is reasonably convenient.

• LL's implied covenant of quiet enjoyment - T has right to quiet use and enjoyment without interference from the LL (in both residential and commercial lease ks). - Is breached by wrongful or constructive eviction a. Breach by wrongful eviction: exclusion from whole or part of the premises 1) Ex - T is partially/constructively evicted if their roof falls down in part of their house. b. Breach by constructive eviction: LL renders premises unsuitable by occupancy. In this case, must est three elements: 1) SI - Substantial interference (Chronic/permanent problem) 2) N - Notice (T must notify L) 3) G - goodbye/get out (T must vacate) • LL's implied warranty of habitability - Applies to residential leases only - Premises must be fit for basic human habitation - Standard: case law and housing code. - Cannot ever waiver this warranty - T's options if LL breaches (M R 3): a. M - move b. R - repair and deduct their costs from future rent c. R - reduce or withhold rent until ct determines the fair value of rent given the premises' condition d. R - remain and seek ds - Distinguish from implied warranty of quiet enjoyment: quiet enjoyment, T MUST vacation. With habitability, T may (but doesn't have to) vacate • Retaliatory eviction: L can't terminate/penalize T in retaliation for T's exercise of legal rights.

• Anti-discrimination laws - FHA: no housing discrimination on basis of race, color, religion, sex, disability, familial status, national origin - Exemptions: a. Owner-occupied buildings with four or fewer units b. Single family homes if owners has no more than 3 - Prohibited actions a. Refusing to negotiate, rent, or sell housing, or give mortgage b. Providing diff terms for sale/rent c. Falsely representing dwelling unavailable - Reasonable accommodations for tenants with disabilities • Caveat lessee (T beware) - In tort, this means LL was under no duty to make the premises safe. - Exceptions: CLAPS a. C - common areas b. L - latent defects (L must warn about hidden defects T couldn't discover by reasonable inspection) c. A - assumption of repairs (L is liable if negligently makes the repair) d. P - public use (short lease for the public use like a museum, liable for a significant defect) e. S - short term lease of a furnished dwelling (like a summer cottage) • Creation of a license: - No writing required - Freely revocable at will of licensor

• Requirements for the burden of a covenant to run with the land: WITHN - W - writing - I - intent - T - touch and concern (= when it affects the parties in their legal roles as landowners, not as members of the community at large) - H - horizontal and vertical privity a. Horizontal privity: nexus between og promising parties (succession of estate) b. Vertical privity: nexus between successor and og covenanting party (non-hostile) - N - notice • Requirements for benefit to run: WITV - W - writing - In - intent - T - touch and concern - V - vertical privity

• Equitable servitude: is a promise that equity will enforce against successors of burdened land (regardless of whether it runs with the land at law) - Creation: WITNESS a. W - writing b. I - intent c. T - touch and concern d. N - notice e. ES - equitable servitude - Common scheme doctrine a. Scheme of development (Including D's lot) when sales began b. D had notice of promise when they took - Doctrine of changed conditions

• What future estate accompanies FSD: possibility of reverter • Fee simple subject to condition subsequent: use of conditional language PLUS an explicit right for the grantor to re-enter. There is NO automatic termination (grantor's prerogative). • Fee simple subject to an executory interest: "To A, but if x event occurs, then to B." Third party, not grantor, takes if condition betrayed. Forfeiture is automatic. • Rules of construction: words of mere desire, hope, intention do't create a defeasible fee. Second rule: absolute restraints on alienation are void. • Life estate: must be measured in explicit lifetime terms (NOT by a term of years). "To A for life" (A = life tenant). • Life estate pur autre vie: LE measured by a life other than the grantee's. ex: To A for the life of B. • The life tenant's rights and duties - Right to all ordinary uses and profits from the land. - Duty to not commit waste (voluntary, permissive, or ameliorative) - Depleting natural resources is generally seen as voluntary waste. Exceptions: if necessary for maintenance on the estate. Another ex: the land is suitable only for exploitation (like if the land was an orchard) - Also note the open mines doctrine - LE has to pay interest on any outstanding mortgages, but not principal. Not obliged to insure the premises for the interest of the future interest holders. - Exception to their duty to not commit ameliorate waste: 1) the change must not adversely affect market value for purposes of the future interest holders' interest; and either 2) future interest holders didn't object, or 3) the premises are useless unless and until the particular change at issue occurs

• Future interests in the grantor: - Possibility of reverter: accompanies a FSD - Right of entry/right of termination: accompanies FS subject to condition subsequent - Reversion: accompanies when grantor conveys estate of lesser duration (other than above) • Future interests in third parties: remainders - Possession on natural expiration of preceding estate (Eg, term of years tenant or life estate). "To A for life, then to B" (A = life tenant; B = remainderman) - Contingent: taker as yet is unascertainable or subject to a condition precedent • Vested remainders: - Indefeasibly vested: certain to acquire, no strings; remainder man is known. - Vested subject to complete defeasance: subject to a condition subsequent. - Need to distinguish condition precedent from condition subsequent: a. Precedent: appears before language introducing you to remainderman. b. Subsequent: appears after language that, alone and set off by commas, creates remainder. - Vested remainder subject to open: class of takers, at least one of which is already qualified to take. a. When does a class close? Rule of convenience: when any member can demand possession.

Inquiry notice means that a subsequent grantee is held to have knowledge of any facts that a reasonable inquiry would have revealed, even if he made no inquiry. In the majority of states, grantees are not charged with inquiry notice from the mere fact that a quitclaim deed was used.

• If an adverse possessor uses land in violation of a recorded real covenant for the limitations period, she takes title free of the real covenant. The nature of the title obtained through adverse possession depends on the occupier's activities on the land. If an adverse possessor uses the land in violation of a real covenant (i.e., a written promise to do or refrain from doing something on the land), she takes title free of the covenant EVEN IF she had knowledge of it. However, if she complies with the covenant for the statutory period, she takes title subject to the real covenant. In either case, if an adverse possessor uses land for the limitations period, she DOES take title to the land.

Fixtures are items so affixed to land that they become part of the realty a. Constructive annexation - items not physically attached to land are fixtures if they're so uniquely adapted to the real estate that it makes no sense to separate them (eg, keys to doors) 2. Common ownership cases - landowner brings chattel onto land a. Annexor's objective intent determines whether items are fixtures 3. Divided ownership cases - landowner doesn't bring chattel onto land a. Item's owner can remove it only if this wouldn't leave unrepaired damage to the premises

• JT interest isn't devisable/capable of being passed down by will • A JT may convey her interest secretly and break the JT, even without the others' knowledge or consent. Buyer will be a TIC. The remaining JTs will stay JTs in respect to each other. • Severance of JT - Sale: JT sells/transfers during lifetime - Partition a. Voluntary: amicable end b. Judicial: in kind (physical division); forced sale (division of proceeds) - Mortgage a. Maj of states (lien theory): no severance b. Minority of states (title theory): severance • In the case of divorce, tenants by the entirety becomes a tenancy in common. • TIC: no right of survivorship. Co-tenant owns individual part and the right to possess the whole. Is devisable, descendible, alienable. • Rights and duties of co-tenants - Possession: no ouster (wrongful exclusion from part/the whole of the property). Each gets to enjoy the whole - Rents and profits: none from co-tenant in exclusive possession (unless ouster) Aka, the person who exclusively is on the land doesn't have to pay rent unless they've committed an ouster. Fair share if leased to third party. - Adverse possession: not unless ouster - Carrying costs (taxes and mortgage interest payments): each pays fair share - Repairs: contributions for reasonable, necessary repairs with notice - Unilateral improvements: no contribution (improver gets a credit at partition equal to any value increase he caused. Gets a debit for any decrease in value his enterprise created)

Every lease contains an implied covenant that neither the landlord nor someone with paramount title will interfere with the tenant's quiet enjoyment and possession of the premises. This covenant is breached by the tenant's total or partial actual eviction from the leased premises. Total actual eviction occurs when the landlord or a paramount title holder excludes the tenant from the entire leased premises. This terminates the tenant's obligation to pay rent. Partial actual eviction occurs when tenant is excluded from only part of the leased premises. Partial eviction by the LL relieves tenant of the obligation to pay rent for the entire premises, even though tenant continues in possession of the remainder of the premises. Partial eviction by a paramount title holder results in an apportionment of rent; i.e., the tenant is liable for the reasonable rental value of the portion that he continues to possess. A paramount title holder's taking possession of an unused barn constitutes partial actual eviction. Thus, rent will be apportioned. If the landlord or a paramount title holder obtains a judgment in an ejectment action against the tenant, the total actual eviction terminates the tenant's obligation to pay rent. If the landlord takes possession of an unused barn on the leased premises and stores farm equipment in it, the partial actual eviction by the landlord terminates the tenant's obligation to pay rent for the entire premises.

• Partial actual eviction by the landlord relieves the tenant of the obligation to pay rent for the entire premises. Every lease contains an implied covenant that neither the landlord nor someone with paramount title will interfere with the tenant's quiet enjoyment and possession of the premises. This covenant is breached by the tenant's total or partial actual eviction from the leased premises. Partial actual eviction occurs when tenant is excluded from only part of the leased premises. Even though the tenant continues in possession of the remainder of the premises, partial eviction by the landlord relieves tenant of the obligation to pay rent for the entire premises rather than for only the portion of the premises from which he was evicted. In contrast, partial eviction by a paramount title holder results in an apportionment of rent; i.e., it relieves the tenant of the obligation to pay rent NOT for the entire premises, but only for the portion of the premises from which he was evicted. The tenant remains liable for the reasonable rental value of the portion that he continues to possess. • The doctrine of constructive eviction provides that where a landlord does an act or fails to perform some service that he has a legal duty to provide, and thereby makes the property uninhabitable, the tenant may terminate the lease and seek damages. However, a tenant cannot claim a constructive eviction unless: (i) the injurious acts were caused by the landlord, (ii) the premises are uninhabitable, and (iii) the tenant vacates the premises within a reasonable time.

The general rule is that a life tenant is responsible for the real estate taxes on the property and interest on the principal to the extent of income from the property. However, a life tenant is liable to the holder of the future interest for property taxes only to the extent of rents received, or the fmv of the property if the life tenant is occupying the property. Aka, if the life tenant isn't occupying the property, and not renting the property, then the future interest holder upon coming into present possession of the property will be unable to sue the former life tenant for unpaid taxes on the property.

• Present and Future Interests in Land I. Overview A. estates in land are possessory interests. they can be possessory now (present estates), or may become possessory in the future (future interests). they may be freeholds, which give possession under some legal title or right to hold (such as fees or life estates), or nonfreeholds, which give mere possession (such as leases). II. The present possessory estates A. A present possessory estate is an interest that gives the holder a right to present possession of the land. B. Fee simple absolute 1. Is freely transferable, devisable by will, and descendible through intestacy. It is absolute ownership of indefinite or potentially infinite duration. 2. Exs: "To A and his heirs;" "To A." 3. Duration: forever 4. Correlative future interest in grantor: none 5. Correlative future interest in third party: none

• Judicial partition - Partition in kind is preferred. However, a forced sale is allowed if a physical division of the property cannot be done. • Tenancy for years: - Known, fixed period of time - Termination automatic (on end date) - No notice needed to terminate - Writing typically needed if for more than one year. • Periodic tenancy: continues for successive intervals until properly terminated. a. Creation: 1) Express ("L to T from month to month") 2) By implication a) No mention of duration but rent at set intervals b) Oral term of years violating SOF c) Holdover tenant after lease ends b. Notice of termination must be given to terminate 1) Common law: at least equal to length of period 2) Month to month: 1 month notice 3) Week to week: 1 week notice 4) Year to year or greater: one month under the restatement

• Tenancy at will - No fixed duration - Terminable at will of either party (to T for as long as L or T desires) • Tenancy at sufferance - Created when T wrongfully holds over past the lease expiration. - L proceeds to recover rent. - Terminates when LL moves to evict or holds T to a new tenancy. • Tenant's duty to repair if the lease is silent: T has duty to maintain the premises. That means they have to make routine repairs. Doesn't have to make repairs that are due to ordinary wear and tear. But T must not commit waste on the property. • Tenant's duty to repair with an express covenant in the lease - Maintain in good repair/condition - T may terminate if premises destroyed without T's fault • Tenant's duty to pay rent - if T breaches and remains on premises, these are options: - Evict - Continue relationship and sue for rent - No self-help by the LL - can't change the locks, take their stuff out of the property - If tenant breaches and is out of possession: a. S - surrender: end lease b. I - ignore: do nothing (and hold T liable for rent) c. R - Relet: new lease (hold T liable for deficiency) • LL's duty to deliver possession - Duty to place T in actual, physical possession

• Statutory redemption is the right of a mortgagor to recover the land after the foreclosure sale has occurred, usually by paying the foreclosure sale price. About half the states provide a statutory right to redeem for some fixed period after the foreclosure sale has occurred, usually six months or one year. The amount to be paid is generally the foreclosure sale price, rather than the amount of the og debt. This right extends to mortgagors and, in some states, to junior lienors. In contrast, equitable redemption is the right of a mortgagor to recover the land by paying the amount overdue on the mortgage, plus interest, at any time before the foreclosure sale. If the mortgagor has defaulted and the mortgage or note contained an acceleration clause, then the full balance of the mortgage must be paid in order to redeem in equity. • A mortgagor can't exercise her statutory right of redemption before default, before the foreclosure sale or only during the foreclosure sale. In the states that provide it, the statutory right to redeem exists for a fixed period after the foreclosure sale. By contrast, all js recognize the mortgagor's right of redemption in equity, which exists until the date of sale and is cut off by foreclosure. A mortgagor may purchase the land at the foreclosure sale, but a statutory right of redemption provides a grace period after foreclosure when the mortgagor may redeem the property.

• When a grantee assumes the mortgage, the grantee expressly promises the grantor-mortgagor that he will pay the mortgage obligation as it becomes due. The mortgagee then becomes a TPB of the grantee's promise to pay and can sue the grantee directly if the grantee fails to pay. After the assumption, the grantor-mortgagor becomes a surety who is secondarily liable to the mortgagee on the note if the grantee fails to pay. The landowner and the attorney are jointly liable, even though the attorney is primarily liable and the landowner is secondarily liable as a surety. • Proceeds from the foreclosure sale are used to satisfy the loan that was foreclosed first. • Generally, the priority of a mortgage is determined by the time it was placed on the property, and the proceeds of a foreclosure sale will be used to pay off the mortgages in the order of their priority. However, if the landowner enters into a modification agreement with the senior mortgagee, raising its interest rate or otherwise making the agreement more burdensome, the junior mortgage will be given priority over the modification. Thus, if the first mortgage debt is larger because of the modification, the second mortgage gains priority over the increase in the debt.

• Executory interests - Take effect by cutting short the interest of a prior taker - Shifting: follows defeasible fee, cuts short someone other than grantor - Springing: cuts short the grantor. • Four step technique for RAP problems: - Determine the future interest - What has to happen for future interest holder to take - Find a measuring life - When will we know if the future interest holder can take • Ask: will we know for sure within 21 years of the relevant lifetime if that interest holder is certain to take or not take • An executory interest with no time limit violates the RAP • RAP won' apply to executory interests in the grantor • a covenant does not run with the land or bind successive owners as an equitable servitude unless there is a benefited estate. Here, there is no such estate, since it appears that the mansion is the only property owned by the landowner in the neighborhood. Therefore, if the buyer of the property resold it, the new owner would not be bound by the covenant in equity. Second, a covenant would only be effective in accomplishing the landowner's purpose if a court of equity specifically enforced it. If there is a significant change in circumstances between the time the easement was granted and the time when the owner wanted to tear the property down, a court might deny specific enforcement of the covenant and enforce it only by way of money ds. • Under the title theory, title is in the mortgagee (lender) until the mortgage has been satisfied or foreclosed. Thus, the mortgagee is entitled to possession on demand at any time

• When a k doesn't say time is of the essence, a seller is required to provide marketable title within a reasonable time after the scheduled closing date. If they do so, buyer has no grounds for refusing to perform. • Shelter rule: a BFP that wins the race/race notice req cuts off the right that any earlier interest holder had in the property; subsequent donees of that BFP will prevail over those earlier interests • If a landowner does not commence an action to eject a would-be adverse possessor before the statute of limitations expires, she's barred from suing for ejectment, and title vests in the possessor. However, the statute of limitations does not run against the holder of a future interest (e.g., remainder, reversion) until her interest becomes possessory. The future interest holder has no right to possession until the prior present estate terminates, and thus no cause of action for ejectment accrues until that time.

• If a mortgage exists on property when a real estate contract is signed, title may be marketable. Every land sale contract contains an implied covenant that the seller will furnish marketable title on the date of closing. Generally, encumbrances (i.e., mortgages, liens, easements, and covenants) render title unmarketable. However, a seller has the right to satisfy a mortgage or lien at the closing with sale proceeds. Thus, if the purchase price is sufficient and this is accomplished simultaneously with the transfer of title, the buyer cannot claim that the seller's title is unmarketable. If a mortgage exists on property when a real estate contract is signed, the mortgage is NOT extinguished. Rather, the mortgage will remain on the land and will encumber the title in the hands of the buyer unless it is satisfied as explained above. If the mortgage is not timely satisfied, the seller will breach the implied covenant of marketability, for which the buyer may pursue several remedies (e.g., rescission, damages, or specific performance with abatement of the purchase price). The contract is NOT void.

• When a seller of land breaches the implied covenant of marketable title, the closing date may be extended to allow the seller time to cure. Every land sale contract contains an implied covenant that the seller will provide marketable title at closing. Marketable title is title reasonably free from doubt, which a reasonably prudent buyer would accept. While it need not be perfect title, it must not present the buyer with an unreasonable risk of litigation. Generally, this means an unencumbered fee simple with good record title. If the buyer determines that the seller's title is unmarketable, he must notify the seller and give her a reasonable time to cure, even if this requires extending the closing date, and even if time is of the essence. When a seller of land breaches the implied covenant of marketable title, rescission IS available as a remedy. If the seller fails to cure the defects, the buyer may rescind the k, sue for ds for breach, get specific performance with abatement of the purchase price, or(in some js) require the seller to quiet title. A ct also may order rescission before the delivery date of an installment land k if the buyer shows that the seller can't possibly cure the defects in time. In contrast with the buyer's remedies, seller CANNOT obtain specific performance or ds (unless the seller cures the title defect within a reasonable time). • When a buyer has a remedy of specific performance in a land sale contract, a court of equity will order a seller to convey the title if the buyer tenders the purchase price. If the seller cannot provide marketable title under the terms of the k, but the buyer wishes to proceed with the transaction, the buyer can usually get specific performance with an abatement of the purchase price in an amount reflecting the title defect. A defect as to the quantity of land conveyed is usually corrected by a pro rata abatement of the price. • Marketable title is title reasonably free from doubt, which generally means free from encumbrances and with good record title. Easements are generally considered encumbrances that render title unmarketable; so if an easement is not provided for in the contract, it usually renders the seller's title unmarketable. There is an exception, however. A majority of courts have held that a beneficial easement that was visible or known to the buyer does not constitute an encumbrance.

• Recording statutes only protect BFP. Requirements to be a BFP: 1) must be a purchaser or creditor; 2) take without actual, constructive, or inquiry notice; 3) pay valuable consideration. • Vertical privity between the covenantor and their successor in interest is not required for the burden of an equitable servitude to run to successors in interest. An equitable servitude is a covenant (that is, a promise to do or not to do something on the land) that, regardless of whether it runs with the land at law, can be enforced in equity against successors of the burdened land unless the successor is a bona fide purchaser (meaning, a subsequent purchaser for value without notice of the covenant). A subsequent purchaser for value of land burdened by a covenant is not bound by it in equity unless they had notice of the covenant when they acquired the land. Note that in most states, successors of the burdened land who are not purchasers (for example, donees) are bound by the covenant whether or not they had notice. Horizontal privity between the original covenanting parties and vertical privity between the covenantor and their successor in interest are not required.

• When a subdivision is created with similar covenants in all deeds, there is a mutual right of endorsement (each lot owner can enforce against every other lot owner) if two things are satisfied: (i) a common scheme for development existed at the time that sales of parcels in the subdivision began; and (ii) there was notice of the existence of the covenant to the party sued.

• Actual possession of a portion of a unitary tract of land is sufficient adverse possession as to give title to the whole of the tract of land after the statutory period, as long as there is a reasonable proportion between the portion actually possessed and the whole of the unitary tract, and the possessor has color of title to the whole tract. Color of title is a document that purports to give title, but for reasons not apparent from its face does not. Usually, the proportion will be held reasonable if possession of the portion was sufficient to put the owner or community on notice of the fact of possession. • For a disability, such as status as a minor, to stop the clock, the disability must have been in existence on the day the adverse possession began. • Anytime someone enters possession with an invalid deed, the possession is hostile bc they're claiming rights superior to those of the true owner • A life tenant must pay interest on any encumbrance on the land to the extent of income or profits derived from the land, but she does not have to pay principal. The remaindermen must pay the principal to protect their interest in the property.

• a junior mortgagee must be named as a party to a senior mortgagee's foreclosure action because it has the right to pay off the senior mortgage to avoid being wiped out by foreclosure. Foreclosure destroys interests ( e.g., liens, mortgages, leases, easements) junior to the mortgage being foreclosed. Thus, if a senior mortgage is in default, a junior mortgagee has the right to pay it off (i.e., redeem it) to avoid being wiped out by its foreclosure. Failure to join the junior mortgagee results in the preservation of its interest despite foreclosure and sale. In contrast, those with interests senior to that of the foreclosing party are not necessary parties bc their interests are not affected by foreclosure. The buyer at the sale takes subject to senior interests, which remain on the land. As explained above, NOT all those with liens on the property are necessary parties to a foreclosure action. Only those with interests subordinate to that of the foreclosing party must be named in the foreclosure action. Failure to name a senior interest holder does not affect that party's interest. Because foreclosure extinguishes interests junior to the mortgage being foreclosed, as explained above, junior mortgagees are necessary parties to a senior mortgagee's foreclosure action.


Kaugnay na mga set ng pag-aaral

Chapter 47: Management of Patients With Intestinal and Rectal Disorders, Prep U--Ch. 47: Mgmt of Patients With Intestinal and Rectal Disorders

View Set

Chapter 7: Entrepreneurship and Starting a Small Business

View Set

Health assessment exam 3 Fall 2023

View Set